OB Exam 2

Réussis tes devoirs et examens dès maintenant avec Quizwiz!

A fetus is assessed at 2 cm above the ischial spines. How would the nurse document the fetal station? -2 +4 0 +2

-2

Which cardinal movement of delivery is the nurse correct to document by station? Extension Flexion Internal rotation Descent

Descent

The nurse is assisting with a G2P1, 24-year-old client who has experienced an uneventful pregnancy and is now progressing well through labor. Which action should be prioritized after noting the fetal head has retracted into the vagina after emerging? Apply pressure to the fundus. Use McRoberts maneuver. Use Zavanelli maneuver. Attempt to push in one of the fetus's shoulders.

Use McRoberts maneuver.

WHen explaining to a pregnant woman about HIV infection and transmission, which information would the nurse include? a. it primarily occurs when there is a large viral load in the body b. HIV is most commonly transmitted via sexual contact c. it affect the majority of infants of mothers with HIV infection d. nurses are most frequently affected by needlesticks

b. HIV is most commonly transmitted via sexual contact

The nurse assesses a client in labor and finds that the fetal long axis is longitudinal to the maternal long axis. How should the nurse document this finding? presentation attitude lie position

lie

The five "Ps" of labor are: passenger, position, powers, presentation, psych. passenger, posture, position, presentation, psych. passenger, position, presentation, pushing, psych. passageway, passenger, position, powers, psych.

passageway, passenger, position, powers, psych.

The most intense time during labor is during the a. latent phase b. active phase c. membranes breaking d. placental expulsion phase

a. latent phase

Which should the nurse identify as a risk associated with anemia during pregnancy? preterm birth newborn with an enlarged liver newborn with heart problems fetal asphyxia

preterm birth

Which observation would suggest that placental separation is occurring? a. uterus stops contracting altogether b. umbilical cord pulsations stop c. uterine shape changes to globular d. maternal blood pressure drops

c. uterine shape changes to globular

Which of these activities would best help the postpartum nurse provide culturally sensitive care for the childbearing family? a. taking a transcultural course b. caring for only families of the nurse's cultural origin c. teaching western beliefs to culturally diverse families d. educating themselves about diverse cultural practices

d. educating themselves about diverse cultural practices

A nurse is providing care to a postpartum woman. The woman gave birth vaginally at 2 a.m. The nurse would anticipate the need to catheterize the client if she does not void by which time? 3:30 a.m. 5:15 a.m. 9:00 a.m. 7:45 a.m.

9:00 a.m.

The nurse is assessing Ms. Smith, who gave birth to her first child 5 days ago. What findings would the nurse expect? a. cream-colored lochia; uterus above the umbilicus b. bright red lochia with clots, uterus two finger-breadths below umbilicus c. light pink or brown lochia; uterus four to five fingerbreadths below umbilicus . yellow, mucousy lochia; uterus at the level of the umbilicus

Light pink or brown lochia; uterus 4 to 5 fingerbreadths below umbilicus

A client has come to the office for her first postpartum visit. On evaluating her blood work, the nurse would be concerned if the hematocrit is noted to have: acutely increased. acutely decreased. slightly increased. slightly decreased.

acutely decreased.

A client is giving birth when shoulder dystocia occurs in the fetus. The nurse recognizes that which condition in the client is likely to increase the risk for shoulder dystocia? nullipara pendulous abdomen preterm birth diabetes

diabetes

A woman has presented to the emergency department with symptoms that suggest an ectopic pregnancy. Which finding would lead the nurse to suspect that the fallopian tube has ruptured? nausea vaginal spotting referred shoulder pain breast tenderness

referred shoulder pain

While visiting a client at home on the 10th day postpartum, the nurse assesses the client's lochia. Which color would the nurse expect the lochia to be? red yellowish white yellowish pink pink

yellowish white

The nurse is caring for a client at 39 weeks' gestation who is noted to be at 0 station. The nurse is correct to document which? The fetus is floating high in the pelvis. The fetus has descended down the birth canal. The client is fully effaced. The fetus is in the true pelvis and engaged.

The fetus is in the true pelvis and engaged.

The nurse would expect a postpartu woman to experience lochia in which sequence? a. rubra, alba, serosa b. rubra, serosa, alba c. serosa, alba, rubra d. alba, rubra, serosa

b. rubra, serosa, alba

Which activity would the nurse include in the teaching plan for parents with a newborn and an older child to reduce sibling rivalry when the newborn is brought home? a. punishing the older child for bedwetting behavior b. sending the sibling to the grandparents house c. planning a daily "special time" for the older sibling d. allowing the sibling to share a room with the infant

c. planning a daily "special time" for the older sibling

A nurse is teaching a nulliparous woman at 42 weeks' gestation about labor induction, which is being recommended by her health care provider. The nurse determines that the woman needs additional teaching when she identifies which assessment as being done before induction? amniotic fluid studies fetal dating maneuvers for fetal positioning Bishop scoring

maneuvers for fetal positioning

A pregnant woman has developed varicosities. Which statement would suggest she needs additional health teaching? "I dorsiflex my feet and ankles frequently." "I maintain a high fluid intake." "I'll try not to stand for long periods." "I wear knee-highs rather than pantyhose."

"I wear knee-highs rather than pantyhose."

Following an initial prenatal visit, a woman's rubella titer results were less than 1:8. On her next visit, the woman asks what this test result means. Which is the best answer to this woman's question about her rubella titer results? "You have immunity against rubella." "You were exposed to someone with rubella." "You recently were infected with rubella." "You are susceptible to a rubella viral invasion."

"You are susceptible to a rubella viral invasion."

Which nursing action prevents a complication associated with the lithotomy position for the birth of the fetus? Providing a paper bag Massaging the client's lower back Placing a wedge under the hips Rubbing the client's legs

Placing a wedge under the hips

The pain of labor is influenced by many factors. What is one of these factors? The woman has a high threshold for pain. The woman has lots of visitors during labor. The woman has a high tolerance for pain. The woman is prepared for labor and birth.

The woman is prepared for labor and birth.

There has been much research done on pain and the perception of pain. What is the result of research done on levels of satisfaction with the control of labor pain? Women report higher levels of satisfaction when different types of relaxation techniques are used to control pain. Women report higher levels of satisfaction when the primary care provider makes the decision on what type of pain control to use. Women report higher levels of satisfaction when regional anesthetics are used to control pain. Women report higher levels of satisfaction when they felt they had a high degree of control over the pain experience.

Women report higher levels of satisfaction when they felt they had a high degree of control over the pain experience.

The nurse is monitoring a client who is in labor and notes the client is happy, cheerful, and "ready to see the baby." The nurse interprets this to mean the client is in which stage or phase of labor? stage two active latent stage three

latent

The nurse is monitoring a pregnant client and notes: contractions causing urge to push, strong intensity, cervix 10 cm, 100% effaced, fetal head crowns when client pushes. The nurse determines the client is currently in which stage or phase of labor? second active third latent

second

Prioritize the postpartum mother's needs 4 hours after giving birth by placing a number 1, 2, 3, or 4 in the blank before each need. _________ Learn how to hold and cuddle the infant. _________Watch a baby bath demonstration given by the nurse. _________ Sleep and rest without being disturbed for a few hours. _________ Interaction time (first 30 minutes) with the infant to facilitate bonding

1. Interaction time (first 30 minutes) with the infant to facilitate bonding 2. Sleep and rest without being disturbed for a few hours 3. Learn how to hold and cuddle the infant 4. Watch a baby bath demonstration given by the nurse

A low-risk client is in the active phase of labor. The nurse evaluates the fetal monitor strip at 10:00 a.m. and notes the following: moderate variability, FHR in the 130s, occasional accelerations, and no decelerations. At what time should the nurse reevaluate the FHR? 10:30 a.m. 11:15 a.m. 11:30 a.m. 10:05 a.m.

10:30 a.m.

A woman with diabetes is in labor. To promote optimal outcomes for the mother and neonate, the nurse monitors the client's blood glucose level closely ensuring that it is maintained below which level? 150 mg/dl 130 mg/dl 120 mg/dl 110 mg/dl

110 mg/dl

A pregnant client with type 1 diabetes is in labor. The client's blood glucose levels are being monitored every hour and she has a prescription for an infusion of regular insulin as needed based on the client's blood glucose levels. Her levels are as follows: 1300: 105 mg/dL (5.83 mmol/L) 1400: 100 mg/dL (5.55 mmol/L) 1500: 120 mg/dL (6.66 mmol/L) 1600: 106 mg/dl (5.88 mmol/L) Based on the recorded blood glucose levels, at which time would the nurse likely administer the regular insulin infusion? 1600 1400 1500 1300

1500

A woman comes to the clinic for her first prenatal visit. As part of the assessment, the woman is screened for rubella antibodies. The nurse determines that a client has immunity against rubella based on which rubella titer? 1:8 1:6 1:0 1:4

1:8

A nurse is providing care to a postpartum woman. Documentation of a previous assessment of a woman's lochia indicates that the amount was moderate. The nurse interprets this as reflecting approximately how much? 25 to 50 ml 10 to 25 ml Over 50 ml Under 10 ml

25 to 50 ml

A woman is to undergo labor induction. The nurse determines that the woman requires cervical ripening if her Bishop score is: 5. 6. 9. 7.

5.

A nurse is providing care to a woman who is 6 hours postpartum. The nurse suspects urinary retention based on which finding? moderate amount of lochia rubra 50 to 70 mL urine per void every hour fundus at the level of the umbilicus urine clear yellow in color

50 to 70 mL urine per void every hour

On a routine home visit, the nurse is asking the new mother about her breastfeeding and personal eating habits. How many additional calories should the nurse encourage the new mother to eat daily? 750 additional calories per day 500 additional calories per day 1,000 additional calories per day 250 additional calories per day

500 additional calories per day

The nurse tests the pH of fluid found on the vaginal exam and determines that the woman's membranes have ruptured based on which result? 6.0 6.5 5.5 5.0

6.5

Why is it important for the nurse to thoroughly assess maternal bladder and bowel status during labor? If the woman's bladder is distended, it may rupture. A full rectum can cause diarrhea. A full bladder or rectum can impede fetal descent. If the woman has a full bladder, labor may be uncomfortable for her.

A full bladder or rectum can impede fetal descent.

The nursing instructor is conducting a class exploring the various changes that occur in the early postpartum period. The instructor determines the session is successful when the students correctly point out which definition of bonding? The skin-to-skin contact that occurs in the birth room Family growing closer together after the birth of a new baby An ongoing process in the year after birth A process of developing an attachment and becoming acquainted with each other

A process of developing an attachment and becoming acquainted with each other

The nurse is reviewing the charts of a group of clients. Which client's weight gain should the nurse be concerned about? the client who gained 25 lb (11 kg) and is in the third trimester the client who gained 8 lb (4 kg) in the first trimester the client who gained 15 lb (7 kg) and is in the second trimester the client who gained 4 lb (2 kg) in the first trimester

the client who gained 8 lb (4 kg) in the first trimester

It has been 8 hours since a woman gave birth vaginally to a healthy newborn. When assessing the woman's fundus, the nurse would expect to find it at: 1 cm below the umbilicus. the level of the umbilicus. between the umbilicus and symphysis pubis. 2 cm below the umbilicus.

the level of the umbilicus.

The nurse is caring for a pregnant client who has condylomata acuminata (anogenital warts) as a result of HPV infection. The nurse should educate the client about: the need to discuss surgical options with her care provider. topical treatments. the importance of hygiene in preventing exacerbations. antibiotic treatments that are safe for the fetus.

the need to discuss surgical options with her care provider.

During the second day postpartum, a nurse notices that a client is initiating breastfeeding with her infant and changing her infant's diapers with some assistance from her partner. Which phase does the nurse recognize that the woman is experiencing? the taking-hold phase the binding-in phase the taking-in phase the letting-go phase

the taking-hold phase

A primigravida 21-year-old client at 24 weeks' gestation has a 2-year history of HIV. As the nurse explains the various options for delivery, which factor should the nurse point out will influence the decision for a vaginal birth? prophylactic antiretroviral therapy (ART) to the infant at birth the viral load the mother's age amniocentesis results at 34 weeks' gestation

the viral load

The client, who has just been walking around her room, sits down and reports leg tightness and achiness. After resting, she states she is feeling much better. The nurse recognizes that this discomfort could be due to which cause? normal response to the body converting back to prepregnancy state infection thromboembolic disorder of the lower extremities hormonal shifting of relaxin and estrogen

thromboembolic disorder of the lower extremities

A client in the third stage of labor has experienced placental separation and expulsion. Why is it necessary for a nurse to massage the woman's uterus briefly until it is firm? to reduce boggy nature of the uterus to remove pieces left attached to uterine wall to lessen the chances of conducting an episiotomy to constrict the uterine blood vessels

to constrict the uterine blood vessels

A young woman experiencing contractions arrives at the emergency department. After examining her, the nurse learns that the client is at 33 weeks' gestation. What treatment can the nurse expect this client to be prescribed? bronchodilators anti-anxiety therapy tocolytic therapy muscle relaxants

tocolytic therapy

A pregnant client at 28 weeks' gestation in preterm labor has received a dose of betamethasone IM today at 1400. The client is scheduled to receive a second dose. At which time would the nurse expect to administer that dose? today at 2200 tomorrow at 1400 tomorrow at 0800 tomorrow at 1800 tomorrow at 1200

tomorrow at 1400

The nurse who is working with parents and their newborn encourages which action to assist the bonding and attachment between them? looking talking feeding touching

touching

A multigravid client has been in labor for several hours and is becoming anxious and distressed with the intensity of her frequent contractions. The nurse observes moderate bloody show and performs a vaginal examination to assess the progress of labor. The cervix is 9 cm dilated. The nurse knows that the client is in which phase of labor? early phase transition phase latent phase active phase

transition phase

The nurse is assessing a primigravida woman who reports vaginal itching, a great deal of foamy yellow-green discharge, and pain during intercourse. The nurse suspects the woman has contracted which disorder? trichomoniasis chlamydia simple yeast infection gonorrhea

trichomoniasis

A nurse is caring for a pregnant client admitted with mild preeclampsia. Which assessment finding should the nurse prioritize? mild hand edema proteinuria of 200 mg/24 hours urine output of less than 15 ml/hr 1+ ankle edema

urine output of less than 15 ml/hr

Which factor puts a client on her first postpartum day at risk for hemorrhage? thrombophlebitis moderate amount of lochia rubra uterine atony hemoglobin level of 12 g/dl (120 g/L)

uterine atony

A nurse is caring for a client who is scheduled to undergo an amnioinfusion. The nurse would question this prescription if which finding is noted upon client assessment? uterine hypertonicity decreased urine output blood pressure of 130/88 mm Hg active genital herpes infection

uterine hypertonicity

A postpartum woman is experiencing subinvolution. When reviewing the client's history for factors that might contribute to this condition, which factors would the nurse identify? Select all that apply. empty bladder breastfeeding early ambulation prolonged labor hydramnios uterine infection

uterine infection prolonged labor hydramnios

A pregnant woman at 36 weeks' gestation comes to the care center for a follow-up visit. The woman is to be screened for group B streptococcus (GBS) infection. When describing this screening to the woman, the nurse would explain that a specimen will be taken from which area(s)? Select all that apply. nasal cavity conjunctiva rectum vagina throat

vagina rectum

A postpartum client who had a cesarean birth reports right calf pain to the nurse. The nurse observes that the client has nonpitting edema from her right knee to her foot. The nurse knows to prepare the client for which test first? transthoracic echocardiogram venogram of the right leg venous duplex ultrasound of the right leg noninvasive arterial studies of the right leg

venous duplex ultrasound of the right leg

A woman comes to the clinic for her first postpartum visit. She gave birth to a healthy term neonate 2 weeks ago. As part of this visit, the woman has a complete blood count drawn. Which result would the nurse identify as a potential problem? white blood cell count 14,000/mm3 (14 ×109/L) hematocrit 42% (0.42) hemoglobin 12.5 g/dL (125 g/L) platelets 350,000/µL (350 ×109/L)

white blood cell count 14,000/mm3 (14 ×109/L)

A nurse performs an initial assessment of a laboring woman and reports the following findings to the primary care provider: fetal heart rate is 152 bpm, cervix is 100% effaced and 5 cm dilated, membranes are intact, and presenting part is well applied to the cervix and at -1 station. The nurse recognizes that the client is in which stage of labor? first, active second third first, latent

within 12 hours of birth

A pregnant client has tested positive for hepatitis B virus. When discussing the situation with the client, the nurse explains that her newborn will be vaccinated with an initial HBV vaccine dose at which time? within 36 hours of birth within 48 hours of birth within 24 hours of birth within 12 hours of birth

within 12 hours of birth

The nurse is analyzing the readout on the EFM and determines the FHR pattern is normal based on which recording? Acceleration of at least 15 bpm for 15 seconds Deceleration followed by acceleration of 15 bpm Increase in variability by 27 bpm Decrease in variability for 15 seconds

Acceleration of at least 15 bpm for 15 seconds

After the birth of the newborn, the mother is ready to be discharged home. The client's mother is present and will remain with her for 1 month. The client's mother tells the nurse that her daughter will not be allowed to leave the house for the first month after the birth, based on the family's cultural customs. How should the nurse respond to this statement? Accept the mother's statement and perform discharge teaching accordingly. Remind the client's mother that the woman needs to get out and get fresh air over the next month. Ask the client's mother why she is putting such restrictions on her daughter. Explain to the client's mother that her daughter may have to go places in caring for the newborn.

Accept the mother's statement and perform discharge teaching accordingly.

A G4P3 client with a history of controlled asthma is upset her initial prenatal appointment is taking too long, making her late for another appointment. What is the nurse's best response when the client insists she knows how to handle her asthma and needs to leave? Remind her to continue taking asthma medications, to monitor peak flow daily, and to monitor the baby's kicks in the second and third trimesters. Schedule an appointment for her to return to discuss her asthma management. Acknowledge her need to leave but ask her to demonstrate the use of inhaler and peak flow meter before she goes; remind her to take regular medications. Note in the chart that the woman was not counseled about her asthma.

Acknowledge her need to leave but ask her to demonstrate the use of inhaler and peak flow meter before she goes; remind her to take regular medications.

A pregnant client is admitted to a health care unit with disseminated intravascular coagulation (DIC). Which prescription is the nurse most likely to receive regarding the therapy for such a client? Administer a ratio of 1 unit of blood to 4 units of frozen plasma. Administer cryoprecipitate and platelets. Give each unit of blood to raise the hematocrit by 3 g/dl (30 g/L). Aim at keeping the client's hematocrit above 20%.

Administer cryoprecipitate and platelets.

A woman is going to have labor induced with oxytocin. Which statement reflects the induction technique the nurse anticipates the primary care provider will prescribe? Administer oxytocin diluted as a "piggyback" infusion. Administer oxytocin in two divided intramuscular sites. Administer oxytocin in a 20 cc bolus of saline. Administer oxytocin diluted in the main intravenous fluid.

Administer oxytocin diluted as a "piggyback" infusion.

A woman at 38 weeks' gestation is in labor and oxytocin is prescribed to augment her labor. When preparing to administer this medication, what action by the nurse would be appropriate? Assist with insertion of a central venous access device for administration. Give the medication as an intramuscular injection using the Z-track technique. Administer the medication piggybacked into a primary IV line using a pump. Give the medication orally every hour for the first 4 hours.

Administer the medication piggybacked into a primary IV line using a pump

A pregnant client is screened for tuberculosis during her first prenatal visit. An intradermal injection of purified protein derivative (PPD) of the tuberculin bacilli is given. Which sign would indicate a positive test result? A flat, circumscribed area under 10 mm in diameter appears in 6 to 12 hours. An indurated wheal over 10 mm in diameter appears in 48 to 72 hours. A flat circumscribed area over 10 mm in diameter appears in 48 to 72 hours. An indurated wheal under 10 mm in diameter appears in 6 to 12 hours.

An indurated wheal over 10 mm in diameter appears in 48 to 72 hours.

A pregnant client is screened for tuberculosis during her first prenatal visit. An intradermal injection of purified protein derivative (PPD) of the tuberculin bacilli is given. Which sign would indicate a positive test result? An indurated wheal under 10 mm in diameter appears in 6 to 12 hours. A flat circumscribed area over 10 mm in diameter appears in 48 to 72 hours. A flat, circumscribed area under 10 mm in diameter appears in 6 to 12 hours. An indurated wheal over 10 mm in diameter appears in 48 to 72 hours.

An indurated wheal over 10 mm in diameter appears in 48 to 72 hours.

A birth room nurse notes that after the provider attempts to remove the placenta, a ball of tissue appears in the woman's vagina accompanied by massive amount of gushing blood. Immediately the woman's vital signs reveal: BP 70/48 mm Hg, pulse rate 150 bpm, and mucous membranes are pale. Which interventions should be the priority for the nurse? Select all that apply. Discontinue the IV oxytocin infusion. Assist the provider in pulling harder on the placenta to get it removed quicker. Apply oxygen mask at 10 L/min. Give the provider assistance by helping reinsert the uterus back through the cervical opening. Call environmental services to mop up the blood.

Apply oxygen mask at 10 L/min. Discontinue the IV oxytocin infusion.

A primigravida client at 39 weeks' gestation calls the OB unit questioning the nurse about being in labor. Which response should the nurse prioritize? Tell the woman to stay home until her membranes rupture. Emphasize that food and fluid should stop or be light. Arrange for the woman to come to the hospital for labor evaluation. Ask the woman to describe why she believes that she is in labor.

Ask the woman to describe why she believes that she is in labor.

The licensed practical nurse is evaluating the tracings on the fetal heart monitor. The nurse is concerned that there is a change in the tracings. What should the LPN do first? Notify the registered nurse. Wait 2 minutes to review another tracing. Assess and reposition the woman. Notify the health care provider.

Assess and reposition the woman.

As a woman enters the second stage of labor, her membranes spontaneously rupture. When this occurs, what would the nurse do next? Assess fetal heart rate for fetal safety. Elevate her hips to prevent cord prolapse. Test a sample of amniotic fluid for protein. Ask her to bear down with the next contraction.

Assess fetal heart rate for fetal safety.

A client's membranes have just ruptured. Her fetus is presenting breech. Which action should the nurse do immediately to rule out prolapse of the umbilical cord in this client? Place the woman in Trendelenburg position. Administer amnioinfusion. Administer oxygen at 10 L/min by face mask. Assess fetal heart sounds.

Assess fetal heart sounds.

During an admission assessment of a client in labor, the nurse observes that there is no vaginal bleeding yet. What nursing intervention is appropriate in the absence of vaginal bleeding when the client is in the early stage of labor? Obtain urine specimen for urinalysis. Monitor hydration status. Monitor vital signs. Assess the amount of cervical dilation (dilatation).

Assess the amount of cervical dilation (dilatation).

A client has just had an epidural placed. Before the procedure, her vital signs were as follows: BP 120/70, P90 bmp, R18 per min, and O2 sat 98%. Now, 3 minutes after the procedure, the client says she feels lightheaded and nauseous. Her vital signs are BP 80/40, P100 bmp, R20 per min, and O2 sat 96%. Which interventions should the nurse perform? Assist the client to the supine position, recheck the blood pressure, and administer an IV bolus of 1000 ml. Assist the client to a sitting position, assess the fetal heart rate, give naloxone, and administer oxygen via face mask. Assist the client to semi-Fowler position, assess the fetal heart rate, start an IV bolus of 500 ml, and administer oxygen via face mask. Assist the client to Trendelenburg position, assess the fetal heart rate, and administer oxygen via face mask.

Assist the client to semi-Fowler position, assess the fetal heart rate, start an IV bolus of 500 ml, and administer oxygen via face mask.

A client has just had an epidural placed. Before the procedure, her vital signs were as follows: BP 120/70, P90 bmp, R18 per min, and O2 sat 98%. Now, 3 minutes after the procedure, the client says she feels lightheaded and nauseous. Her vital signs are BP 80/40, P100 bmp, R20 per min, and O2 sat 96%. Which interventions should the nurse perform? Assist the client to the supine position, recheck the blood pressure, and administer an IV bolus of 1000 ml. Assist the client to semi-Fowler position, assess the fetal heart rate, start an IV bolus of 500 ml, and administer oxygen via face mask. Assist the client to a sitting position, assess the fetal heart rate, give naloxone, and administer oxygen via face mask. Assist the client to Trendelenburg position, assess the fetal heart rate, and administer oxygen via face mask.

Assist the client to semi-Fowler position, assess the fetal heart rate, start an IV bolus of 500 ml, and administer oxygen via face mask.

A G1 P1001 mother is just home after giving birth to her first child 5 days ago. Her birth was complicated by an emergency cesarean birth resulting from incomplete cervical dilation (dilatation) and hemorrhage. The nurse determines that the mother has not slept longer than 3 hours at one time. The appropriate nursing diagnosis for this client care issue is: At risk for safety due to low hemoglobin. At risk for inadequate healing due to decreased nutrition. At risk for interruption of tissue integrity. At risk for postpartum depression due to inadequate rest.

At risk for postpartum depression due to inadequate rest.

During which time is the nurse correct to document the end of the third stage of labor? When the mother is moved to the postpartum unit Following fetal birth At the time of placental delivery When pushing begins

At the time of placental delivery

The nurse would anticipate a cesarean birth for a client who has which active infection present at the onset of labor? a. hepatitis b. herpes simplex virus c. toxoplasmosis d. human papillomavirus

B. Herpes simplex virus

The nurse recognizes that documenting accurate blood pressures is vital in the diagnosing of preeclampsia, severe preeclampsia and eclampsia. The nurse suspects preeclampsia based on which finding? BP of 160/110 mm Hg on two occasions after 28 weeks' gestation BP of 130/90 mm Hg on three occasions 3 hours apart BP of 140/90 mm Hg last week and at current visit after 20 weeks' gestation BP of 120/90 mm Hg on three occasions after 20 weeks' gestation

BP of 140/90 mm Hg last week and at current visit after 20 weeks' gestation

Which occurs as a result of contraction decrement? Select all that apply. Blood flow to the fetus improves. Fetal heart rate should return to baseline. The mother feels the contraction intensifying. The mother feels a gush of water in the perineal area. The fetus is pushed down the birth canal.

Blood flow to the fetus improves. Fetal heart rate should return to baseline.

A shoulder dystocia situation is called in room 4. The nurse enters the room to help and the health care provider says to the nurse, "McRoberts maneuver." What does the nurse do next? Move the client into a hands-and-knees position, to straighten the sacral curve and release the posterior shoulder Push the fetal head back into the uterus and prepare the client for cesarean birth Bring the client's knees back toward the shoulders, causing hyperflexion of the hips and rotation of the pubic symphysis Apply downward pressure above the pubic bone of the client, in an attempt to rotate the anterior shoulder

Bring the client's knees back toward the shoulders, causing hyperflexion of the hips and rotation of the pubic symphysis

A client who is in active labor and whose cervix had dilated to 6 cm experiences a weakening in the intensity and frequency of her contractions and exhibits no further progress in labor. The nurse interprets this as a sign of: a. hypertonic labor b. precipitate labor c. hypotonic labor d. dysfunctional labor

C. Hypotonic labor

The nurse is developing a plan of care for a woman experiencing dystocia. Which nursing intervention would be the nurses highest priority? a. changing the woman's position frequently b. providing comfort measures to the woman c. monitoring the fetal heart rate patterns d. keeping the couple informed of the labor progress

C. Monitoring the fetal heart rate patterns

There are four essential components of labor. The first is the passageway. It is composed of the bony pelvis and soft tissues. What is one component of the passageway? False pelvis Uterus Cervix Perineum

Cervix

The nurse is assessing a 35-year-old woman at 22 weeks' gestation who has had recent laboratory work. The nurse notes fasting blood glucose 146 mg/dl (8.10 mmol/L), hemoglobin 13 g/dl (130 g/L), and hematocrit 37% (0.37). Based on these results, which instruction should the nurse prioritize? Check blood sugar levels daily. the signs and symptoms of urinary tract infection Include iron-enriched foods in the diet. Take daily iron supplements.

Check blood sugar levels daily.

Before calling the health care provider to report a slow progression or an arrest of labor, several assessments need to be made. What other maternal assessment does the nurse need to make prior to calling the health care provider? Make sure the epidural medication is turned down. Make sure the client is lying on her left side. Assess vital signs every 30 minutes. Check for a full bladder.

Check for a full bladder.

The nurse is assessing a client in active labor and notes a small, rounded mass above the symphysis pubis that is distended but nontender. Which action should the nurse prioritize? Assume this is part of the uterus. Ask the client if the mass has always been present. Check the chart for the last void. Notify the health care provider about the mass.

Check the chart for the last void.

A client at 10 weeks' gestation is complaining of ptyalism over the past 2 weeks. What intervention would the nurse recommend to this client? Select all that apply. Suck on hard candies. Wear a panty liner. Eat a large, protein rich meal in the evening. Use saline nasal spray. Chew gum.

Chew gum. Suck on hard candies.

The nurse is documenting the length of time in the second stage of labor. Which data will the nurse use to complete the documentation? Complete cervical dilation (dilatation) and time of fetal birth Time of mucus plug expulsion and full cervical dilation Effacement time and time when contractions are regular Admission time and time of fetal birth

Complete cervical dilation (dilatation) and time of fetal birth

The nurse is documenting the length of time in the second stage of labor. Which data will the nurse use to complete the documentation? Time of mucus plug expulsion and full cervical dilation Effacement time and time when contractions are regular Complete cervical dilation (dilatation) and time of fetal birth Admission time and time of fetal birth

Complete cervical dilation (dilatation) and time of fetal birth

The nurse cares for a pregnant client in labor and determines the fetus is in the right occiput anterior (ROA) position. Which action by the nurse is best? Continue to monitor the progress of labor. Educate the client this fetal position may result in a longer labor. Auscultate fetal heart rate (FHR) in the left upper quadrant. Prepare the client for cesarean birth of the fetus.

Continue to monitor the progress of labor.

The nurse is caring for a client who is a gravida 2 para 1 and had a previous cesarean section. The client has had no complications with the pregnancy and prefers to have this delivery vaginally. Which monitoring system best assesses for the ability to delivery vaginally? Continuous internal monitoring of uterine contractions Intermittent monitoring of the uterine resting tone Intermittent fetal heart rate auscultation Continuous external monitoring of uterine contractions

Continuous internal monitoring of uterine contractions

The nursing instructor is preparing a class discussing the role of the nurse during the labor and birthing process. Which intervention should the instructor point out has the greatest effect on relieving anxiety for the client? Pharmacologic pain management Massage therapy Prenatal classes Continuous labor support

Continuous labor support

The nurse is preparing a young couple for the upcoming birth of their child, and the mother expresses concern for needing pain medications and the effects on the fetus. When counseling the couple about pain relief, the nurse would incorporate which information in the teaching about measures to help to decrease the requests for pain medication? Continuous support through the labor process helps decrease the need for pain medication. A quick epidural can replace the need for pain medication. Sitting in a hot tub helps decrease the need for pain medication. Lying on an ice pack can help decrease the need for pain medication.

Continuous support through the labor process helps decrease the need for pain medication.

When reviewing the medical record of a client the nurse notes that the woman has a condition in which the fetus cannot physically pass through the maternal pelvis. The nurse interprets this as: a. cervical insufficiency b. contracted pelvis c. maternal disproportion d. fetopelvic disproportion

D. Fetopelvic disproportion Fetopelvic disproportion occurs when fetus is too large to pass through the maternal pelvis.

The nurse is assessing a new client who is being admitted with gestational hypertension. Which nursing diagnosis should the nurse prioritize for this client? Decreased reflexes due to medication administration Risk for injury related to fetal distress Deficient fluid volume related to vasospasm of arteries Imbalanced nutrition related to decreased sodium levels

Deficient fluid volume related to vasospasm of arteries

A new mother is in the second developmental stage of becoming a mother and is becoming independent in her actions. Which action by the nurse would best foster this stage? Telling the mother to feed the baby when it cries. Changing the infant's diapers for the mother Correcting the mother when she holds the newborn incorrectly. Demonstrating how to do cord care on the newborn

Demonstrating how to do cord care on the newborn

A client is Rh-negative and has given birth to her newborn. What should the nurse do next? Determine if this is the client's first baby. Ask if the client received rH immunoglobulins during the pregnancy. Determine the newborn's blood type and rhesus. Administer Rh immunoglobulins intramuscularly.

Determine the newborn's blood type and rhesus.

The nurse is monitoring a client in the first stage of labor. The nurse determines the client's uterine contractions are effective and progressing well based on which finding? Engagement of fetus Rupture of amniotic membranes Bloody show Dilation (dilatation) of cervix

Dilation (dilatation) of cervix

The nurse is monitoring a woman who is receiving IV oxytocin to assist with uterine irritability. Which action should the nurse prioritize if the woman's contractions are determined to be 80 seconds in length after 1 hour of administration of the oxytocin? Continue to monitor contraction duration every 2 hours. Slow the infusion to under 10 gtts per minute. Increase the flow rate of the main line infusion. Discontinue the oxytocin infusion.

Discontinue the oxytocin infusion.

A laboring woman is receiving oxytocin IV to augment her labor and 2 hours later begins having contractions every 2 minutes lasting 60 to 90 seconds each with little, if any, rest time in between the contractions. At this time, which interventions would be the priority for the nurse caring for this client? Select all that apply. Ask the woman to drink 32 ounces (1 L) of water. Administer an IV bolus of fluids. Administer betamethasone to mature the fetal lungs. Apply oxygen to the woman via mask at 8 to 10 L/min. Discontinue the oxytocin infusion.

Discontinue the oxytocin infusion. Administer an IV bolus of fluids. Apply oxygen to the woman via mask at 8 to 10 L/min.

At which time in a client's labor process would the nurse encourage effleurage? At home as the client is determining true labor During the early labor phase During the active labor phase Immediately prior to birth

During the early labor phase

The nurse encourages a woman with gestational diabetes to maintain an active exercise period during pregnancy. Prior to this exercise period, the nurse would advise her to take which action? Eat a sustaining-carbohydrate snack. Inject a bolus of insulin. Eat a high-carbohydrate snack. Add a bolus of long-acting insulin.

Eat a sustaining-carbohydrate snack.

A postpartum client is having difficulty stopping her urine stream. Which should the nurse do next? Determine if the client is emptying her bladder. Perform an "in and out" catheter on the client. Educate the client on how to perform Kegel exercises. Ask the client when she last urinated.

Educate the client on how to perform Kegel exercises.

The nurse is preparing a postpartum client for discharge 72 hours after birth. The client reports bilateral breast pain around the entire breast on assessment. The nurse predicts this is related to which cause after noting the skin is intact and normal coloration? Engorgement Excessive oxytocin Mastitis Blocked milk duct

Engorgement

A postpartum mother calls the nurse in and tells her that her right calf hurts whenever she walks around the room or in the hall. What other data needs to be collected in assessing this client for a DVT? Select all that apply. Have the mother actively flex both legs for equal movement. Note any reddened areas on the right calf. Measure the diameter of both calves. Feel the right calf for increased warmth. Note capillary refill of the toes.

Feel the right calf for increased warmth. Note any reddened areas on the right calf. Measure the diameter of both calves.

How does a woman who feels in control of the situation during labor influence her pain? Decreased feeling of control helps during the third stage. Feelings of control are inversely related to the client's report of pain. Feeling in control shortens the overall length of labor. There is no association between the two factors.

Feelings of control are inversely related to the client's report of pain.

In which manner is the fetal status best assessed during the active and transition stages of labor? Fetal kicks over a 1-minute period Fetal heart rate between contractions Fetal heart rate at the peak of a contraction Fetal movement on the tocometer

Fetal heart rate at the peak of a contraction

Which consideration is a priority when caring for a mother with strong contractions 1 minute apart? The station in which the fetus is located Maternal heart rate and blood pressure Fetal heart rate in relation to contractions Maternal request for pain medication

Fetal heart rate in relation to contractions

The nurse is appraising the laboratory results of a pregnant client who is in her second trimester and notes the following: thyroid stimulating hormone (TSH) slightly elevated, glucose in the urine, complete blood count (CBC) low normal, and normal electrolytes. The nurse prioritizes further testing to rule out which condition? Hyperthyroidism Anemia Preeclampsia Gestational diabetes

Gestational diabetes

The nurse has received the results of a client's postpartum hemoglobin and hematocrit. Review of the client's history reveals a prepartum hemoglobin of 14 g/dl (140 g/L) and hematocrit of 42% (0.42). Which result should the nurse prioritize? Hemoglobin 9 g/dl (90 g/L) and hematocrit 32% (0.32) in a woman who has given birth by cesarean Hemoglobin 13 g/dl (130 g/L) and hematocrit 40% (0.40) in a woman who has given birth vaginally Hemoglobin 12 g/dl (120 g/L) and hematocrit 38% (0.38) in a woman who has given birth vaginally Hemoglobin 11 g/dl (110 g/L) and hematocrit 34% (0.34) in a woman who has given birth by cesarean

Hemoglobin 9 g/dl (90 g/L) and hematocrit 32% (0.32) in a woman who has given birth by cesarean

A 24-year-old woman presents with vague abdominal pains, nausea, and vomiting. An urine hCG is positive after the client mentioned that her last menstrual period was 2 months ago. The nurse should prepare the client for which intervention if the transvaginal ultrasound indicates a gestation sac is found in the right lower quadrant? Bed rest for the next 4 weeks Intravenous administration of a tocolytic Immediate surgery Internal uterine monitoring

Immediate surgery

The nurse is assisting a client through labor, monitoring her closely now that she has received an epidural. Which finding should the nurse prioritize to the anesthesiologist? Urinary retention Dry, cracked lips Rapid progress of labor Inability to push

Inability to push

A multigravida client admitted in active labor has progressed well and the client and fetus have remained in good condition. Which action should the nurse prioritize if the client suddenly shouts out, "The baby is coming!"? Time the contractions. Contact the primary care provider. Auscultate the fetal heart tones. Inspect the perineum. SUBMIT ANSWER

Inspect the perineum.

A nurse is assessing a postpartum client. Which measure is appropriate? Wear sterile gloves when assessing the pad and perineum. Instruct the client to empty her bladder before the examination. Perform the examination as quickly as possible. Place the client in a supine position with her arms overhead for the examination of her breasts and fundus.

Instruct the client to empty her bladder before the examination.

Which nursing intervention should the nurse perform when assessing fetal well-being through abdominal ultrasonography in a client? Inform the client that she may feel hot initially. Instruct the client to refrain from emptying her bladder. Instruct the client to report the occurrence of fever. Obtain and record vital signs of the client.

Instruct the client to refrain from emptying her bladder.

The nursing instructor is leading a discussion on the physical changes to a woman's body after the birth of the baby. The instructor determines the session is successful after the students correctly point out which process results in the return of nonpregnant size and function of the female organs? Evolution Involution Decrement Progression

Involution

A pregnant client is admitted to a maternity clinic for birth. The client wishes to adopt the kneeling position during labor. The nurse knows that which of the following is an advantage of adopting a kneeling position during labor? It facilitates vaginal examinations. It helps the woman in labor to save energy. It facilitates external belt adjustment. It helps to rotate the fetus in a posterior position.

It helps to rotate the fetus in a posterior position.

A woman with class II heart disease is experiencing an uneventful pregnancy and is now prescribed bed rest at 36 weeks' gestation by her health care provider. The nurse should point out that this is best accomplished with which position? Lie flat on her back. Stay in high Fowler position. Lie in a semi-recumbent position. Use pillows and wedges to stay in a fully recumbent position.

Lie in a semi-recumbent position.

Which assessment finding 1 hour after birth should be reported to the health care provider? Fundus of uterus is palpable at the level of the umbilicus. Lochia rubra is saturating a pad every 45 to 60 minutes. Fundus is displaced to the right, and bladder is hard. Large, bruised hemorrhoids are protruding from the anal opening.

Lochia rubra is saturating a pad every 45 to 60 minutes.

What two elements play the biggest role in becoming a mother after delivery of her newborn? Confidence and happiness with the pregnancy Interactions with the child and support systems Planned and desired pregnancy and previous experience with infants Love and attachment to the child and engagement with the child

Love and attachment to the child and engagement with the child

The nurse is screening a woman during a home visit following birth. The nurse identifies which risk factors for developing postpartum depression? Select all that apply. Lack of social support Low socioeconomic status Low self-esteem Feeling overwhelmed and out of control Involving family in infant care

Low self-esteem Feeling overwhelmed and out of control Low socioeconomic status Lack of social support

What important instruction should the nurse give a pregnant client with tuberculosis? Avoid red meat. Wear light, cotton clothes. Avoid direct sunlight. Maintain adequate hydration.

Maintain adequate hydration.

Shoulder dystocia is a true medical emergency that can cause fetal demise because the baby cannot be born. Stuck in the birth canal, the infant cannot take its first breath. Which maneuver is first attempted to deliver an infant with shoulder dystocia? McRoberts maneuver McGeorge maneuver McDonald maneuver McRonald maneuver

McRoberts Maneuver

A postpartum client has decided to bottle feed her newborn. After teaching the woman about it, the nurse determines that the teaching was successful based on which client statement(s)? Select all that apply. "I will make sure the nipple and neck of the bottle are filled with formula during a feeding." "I will get my newborn to suck by touching the nipple to the lips." "I will use warm water to mix the powdered formula." "I will be sure not to use the microwave to warm the formula." "I will store any formula left over from a feeding in the refrigerator."

McRoberts maneuver

The nurse is caring for several postpartum clients on the unit. Which client's reaction should the nurse prioritize for possible intervention? Neglects to engage or provide care or show interest in infant. Hesitates to hold newborn, expressing disappointment with baby's appearance. Tearful for several days, difficulty eating and sleeping. Express doubt in ability to care for newborn.

Neglects to engage or provide care or show interest in infant.

General anesthesia is not used frequently in obstetrics because of the risks involved. There are physiologic changes that occur during pregnancy that make the risks of general anesthesia higher than it is in the general population. What is one of those risks? The client is less sensitive to inhalation anesthetics. The client is more sensitive to preanesthetic medications. Neonatal depression is possible. Fetal hypersensitivity to anesthetic is possible.

Neonatal depression is possible.

When caring for a client in the third stage of labor, the nurse notices that the expulsion of the placenta has not occurred within 5 minutes after birth of the infant. What should the nurse do? Do a vaginal exam to see if the placenta is stuck in the birth canal. Increase the IV tocolytic to help in expulsion of the placenta. Nothing. Normal time for stage three is 5 to 30 minutes. Notify the primary care provider of the problem.

Nothing. Normal time for stage three is 5 to 30 minutes.

A client at 25 weeks' gestation presents with a blood pressure of 152/99 mm Hg, pulse 78 beats/min, no edema, and urine negative for protein. What would the nurse do next? Provide health education Notify the health care provider Document the client's blood pressure Assess the client for ketonuria

Notify the health care provider

A pregnant woman diagnosed with diabetes should be instructed to perform which action? Notify the primary care provider if unable to eat because of nausea and vomiting. Prepare foods with increased carbohydrates to provide needed calories. Ingest a smaller amount of food prior to sleep to prevent nocturnal hyperglycemia. Discontinue insulin injections until 15 weeks gestation.

Notify the primary care provider if unable to eat because of nausea and vomiting.

The maternal health nurse is caring for a pregnant client with a history of asthma who requires maintenance medication for the management of the disease. Which action by the nurse best reinforces information provided to the client regarding maintenance of her health? Ask the client to show the nurse deep breathing techniques. Review the client's peak flow meter readings. Observe the client taking her inhaler. Teach the client how to use a spacer with an inhaler.

Observe the client taking her inhaler.

A nurse caring for a pregnant client suspects substance use during pregnancy. What is the priority nursing intervention for this client? Provide education material on cessation of substance use. Determine if the client has emotional support. Obtain a urine specimen for a drug screening. Determine how long the client has been using drugs.

Obtain a urine specimen for a drug screening.

A 26-year-old primigravida has brought her doula to the birthing center for support during her labor and birth. The doula has been helping her through the past 16 hours of labor. The laboring woman is now 6 cm dilated. She continues to report severe pain in her back with each contraction. The client finds it comforting when her doula uses the ball of her hand to put counterpressure on her lower back. What is the likely cause of the woman's back pain? Breech presentation Fetal macrosomia Nongynecoid pelvis Occiput posterior position

Occiput posterior position

A client who has a breastfeeding newborn reports sore nipples. Which intervention can the nurse suggest to alleviate the client's condition? Recommend a moisturizing soap to clean the nipples. Encourage use of breast pads with plastic liners. Fasten nursing bra flaps immediately after feeding. Offer suggestions based on observation to correct positioning or latching.

Offer suggestions based on observation to correct positioning or latching.

A woman presents at Labor and Delivery very upset. She reports that she has not felt her baby moving for the last 6 hours. The nurse listens for a fetal heart rate and cannot find a heartbeat. An ultrasound confirms fetal death and labor induction is started. What intervention by the nurse would be appropriate for this mother at this time? Explain to her that there was probably something wrong with the infant and that is why it died. Recommend that she not hold the infant after it is delivered so as to not upset her more. Call the hospital chaplain to talk to the parents. Offer to take pictures and footprints of the infant once it is delivered.

Offer to take pictures and footprints of the infant once it is delivered.

The nurse is instructing on maternal hormones which may impact the onset of labor. Which hormones are included in the discussion? Select all that apply. Oxytocin Insulin Testosterone Progesterone Prostaglandins Thyroxine

Oxytocin Progesterone Prostaglandins

The nurse is preparing an educational event for pregnant women on the topic of labor pain and birth. The nurse understands the need to include the origin of labor pain for each stage of labor. What information will the nurse present for the first stage of labor? It is reported as the worst pain a woman will ever feel. Diffuse abdominal pain signals a complication with progression of labor. Pain is focal in nature. Pain originates from the cervix and lower uterine segment.

Pain originates from the cervix and lower uterine segment.

A nurse is auscultating the fetal heart rate of a woman in labor. To ensure that the nurse is assessing the FHR and not the mother's heart rate, which action would be most appropriate for the nurse to do? Ask the woman to hold her breath while assessing the FHR. Instruct the woman to bend her knees and flex her hips. Have the woman lie completely flat on her back while auscultating. Palpate the mother's radial pulse at the same time.

Palpate the mother's radial pulse at the same time.

The nurse is caring for a 2-day-old newborn whose mother was diagnosed with cytomegalovirus during the first trimester. On which health care provider prescription should the nurse place the priority? Perform a hearing screen test. Monitor growth and development. Obtain a urine specimen. Assess pulse rate.

Perform a hearing screen test.

A woman with known cardiac disease from childhood presents at the obstetrician's office 6 weeks' pregnant. What recommendations would the nurse make to the client to address the known cardiac problems for this pregnancy? Select all that apply. Plan periods of rest into the workday. Continue taking the scheduled warfarin. Increase the amount of sodium in your diet to compensate for the expanding fluid needs of the fetus. Let the physician know if you become short of breath or have a nighttime cough. Receive pneumococcal and influenza vaccines.

Plan periods of rest into the workday. Receive pneumococcal and influenza vaccines. Let the physician know if you become short of breath or have a nighttime cough.

A pregnant client with diabetes in the hospital reports waking up with shakiness and diaphoresis. Which action should the nurse prioritize after discovering the client's fasting blood sugar is 60 mg/dl (3.33 mmol/L)? Recheck her blood sugar for accuracy. Stay with her, and ask another nurse to bring her insulin. Withhold her insulin, and notify the health care provider. Provide the client some milk to drink.

Provide the client some milk to drink.

The maternal health nurse is caring for a group of high-risk pregnant clients. Which client condition will the nurse identify as being the highest risk for pregnancy? Repaired atrial septal defect Pulmonary hypertension Secondary hypertension Loud systolic murmur

Pulmonary hypertension

Parents tell the nurse that their 3-year-old son has begun to have "accidents" at home following the arrival of his baby sister and wants to sit in his mother's lap all the time now. What advice would the nurse offer these parents? Select all that apply. Be aware of potential aggressive behaviors from the older sibling. Set aside time every day for the parents to focus on the big brother exclusively. Tell the older sibling that he is a big boy and has to share his mommy with the little sister. Buy the older sibling a doll for him to care for, as the mother is caring for the new baby. Scold him whenever he wets his pants and place him back in diapers.

Set aside time every day for the parents to focus on the big brother exclusively. Buy the older sibling a doll for him to care for, as the mother is caring for the new baby. Be aware of potential aggressive behaviors from the older sibling.

A woman who has sickle cell anemia asks the nurse if her infant will develop sickle cell disease. The nurse would base the answer on which information? Sickle cell anemia is not inherited; it occurs following a malaria infection. Sickle cell anemia is recessively inherited. Sickle cell anemia has more than one polygenic inheritance pattern. Sickle cell anemia is dominantly inherited.

Sickle cell anemia is recessively inherited.

A nurse is presenting an in-service program about complications that can arise during labor. The nurse determines that the teaching was successful when the group correctly chooses which findings as suggesting an amniotic fluid embolism? Select all that apply. Maternal tachycardia Sudden onset of respiratory distress Maternal hypotension Acute, continuous abdominal pain Slow onset of fetal distress

Sudden onset of respiratory distress Maternal hypotension Maternal tachycardia

The nurse is caring for a client in active labor. Which assessment finding should the nurse prioritize and report to the team? Unrelieved pain Sudden shortness of breath Bradycardia Bradypnea

Sudden shortness of breath

A 39-year-old multigravida with diabetes presents at 32 weeks' gestation reporting she has not felt movement of her fetus. Assessment reveals the fetus has died. The nurse shares with the mother that the institution takes pictures after the birth and asks if she would like one. What is the best response if the mother angrily says no and starts crying? Tell her that once she gets over her shock and grief, she will probably be happy to have the photos. Tell her that the hospital will keep the photos for her in case she changes her mind. Apologize and tell her that the photos will be destroyed immediately. Console her with the fact that she has other children.

Tell her that the hospital will keep the photos for her in case she changes her mind.

A new mother tells the postpartum nurse that she thinks her baby does not like her since it cries often when she holds it. How should the nurse respond to this statement? Recommend rooming-in to foster attachment and confidence by the mother. Recommend that she talk to the unit social worker to get the mother some counseling prior to discharge. Dismiss the mother's concerns by telling her that you are sure she doesn't really mean it. Tell the mother that it is natural to have feelings of uncertainty when adjusting to a new baby.

Tell the mother that it is natural to have feelings of uncertainty when adjusting to a new baby

The nurse is caring for a client who is late in her pregnancy. What assessment finding should the nurse attribute to the role of prostaglandins? The cervix is dilating The uterus is relaxing The perineum is relaxing The cervix is softening

The cervix is softening

A pregnant client wants to know why the labor of a primigravida usually lasts longer than that of a woman who has already given birth once and is pregnant a second time. What explanation should the nurse offer the client? Braxton Hicks contractions are not strong enough during first pregnancy. Spontaneous rupture of membranes occurs during first pregnancy. Contractions are stronger during the first pregnancy than the second. The cervix takes around 12 to 16 hours to dilate during first pregnancy.

The cervix takes around 12 to 16 hours to dilate during first pregnancy.

A 24-year-old primigravida client at 39 weeks' gestation presents to the OB unit concerned she is in labor. Which assessment findings will lead the nurse to determine the client is in true labor? The client reports back pain, and the cervix is effacing and dilating. The contraction pains have been present for 5 hours, and the patterns are regular. The contraction pains are 2 minutes apart and 1 minute in duration. After walking for an hour, the contractions have not fully subsided.

The client reports back pain, and the cervix is effacing and dilating.

Which psychosocial state is anticipated when the client enters the active phase of labor? The client will become angry and begin to scream. The client will become more talkative and excited about the birth. The client will become more quiet and introverted. The client will become tired and want the process over.

The client will become more quiet and introverted.

Which client outcome during the active phase labor is best? The client will walk in the hall for 15 minutes every 2 hours. The client will practice breathing techniques during contractions. The client will state a pain level of 7 or less during contractions. The client will tolerate 8 oz (240 ml) of clear liquids.

The client will practice breathing techniques during contractions.

A nurse helps a postpartum woman out of bed for the first time postpartum and notices that she has a very heavy lochia flow. Which assessment finding would best help the nurse decide that the flow is within normal limits? The flow contains large clots. Her uterus is soft to your touch. The flow is over 500 mL. The color of the flow is red.

The color of the flow is red.

A pregnant client arrives to the clinic for a prenatal visit appearing uncomfortable. During the assessment, the nurse determines the client is experiencing fairly strong contractions at 12:05 p.m., 12:10 p.m., 12:15 p.m., and 12:20 p.m. What can the nurse conclude from these findings? The duration of the contractions is every 5 minutes. The client is in active labor. The frequency of the contractions is every 5 minutes. The client can be sent home.

The frequency of the contractions is every 5 minutes.

The nurse is looking at the latest lab work for her postpartum client. The client's predelivery hemoglobin and hematocrit (H & H) was 12.8 and 39, respectively. This morning, the client's values are 8.9 and 30. How would the nurse interpret these lab values? The client will be tired, so encourage her to sleep whenever the baby sleeps. These values are expected for a 1-day postpartum mother. The health care provider needs to be notified of the latest lab values. The client will need a transfusion, so the RN needs to be notified.

The health care provider needs to be notified of the latest lab values.

Which statement describes why hypertonic contractions tend to become very painful? More than one contraction may begin at the same time, as receptor points in the myometrium act independently of each other. The myometrium becomes sensitive from the lack of relaxation and anoxia of uterine cells. The number of uterine contractions is very low or infrequent. There is an increase in the length of labor because so many contractions are needed to achieve cervical dilation (dilatation).

The myometrium becomes sensitive from the lack of relaxation and anoxia of uterine cells.

A client experiencing contractions presents at a health care facility. Assessment conducted by the nurse reveals that the client has been experiencing Braxton Hicks contractions. The nurse has to educate the client on the usefulness of Braxton Hicks contractions. Which role do Braxton Hicks contractions play in aiding labor? These contractions help in softening and ripening the cervix. These contractions increase oxytocin sensitivity. These contractions increase the release of prostaglandins. These contractions make maternal breathing easier.

These contractions help in softening and ripening the cervix.

Hypertonic labor is labor that is characterized by short, irregular contractions without complete relaxation of the uterine wall in between contractions. Hypertonic labor can be caused by an increased sensitivity to oxytocin. What would the nurse do for a client who is in hypertonic labor because of oxytocin augmentation? Increase the methotrexate. Increase the oxytocin. Turn off the oxytocin. Turn off the methotrexate.

Turn off the oxytocin.

The nurse is caring for a client who prefers resting on their back during the labor process. To facilitate client wishes, which nursing action is required? Utilize a wedge under one hip. Elevate the knee gatch. Place the tocotransducer low on abdomen. Raise the head of the bed.

Utilize a wedge under one hip.

A woman in her first trimester of pregnancy has noted an increase in a thick, whitish vaginal discharge even though she showers daily. The woman shares this information with the clinic nurse, who provides some client education on the topic of leukorrhea. Which interventions should be addressed in this discussion? Select all that apply. It is normal for the discharge to change in color or odor. Many woman find douching to provide a feeling of cleanliness. Wear cotton underwear during the day. Tampons are the most reliable product to control the flow during the day. A perineal pad to absorb the discharge may help.

Wear cotton underwear during the day. A perineal pad to absorb the discharge may help.

A new mother asks the nurse what she is allowed to do when she goes home from the hospital. Which statement by the nurse would be correct? You need to hire a maid for the first month after delivery to help out around the house. You should not lift anything heavier than your infant in its carrier. Only clean half of the house per day to allow yourself more rest. You should be able to resume normal activities after 2 weeks.

You should not lift anything heavier than your infant in its carrier.

The nurse manager is reviewing all the L & D clients on the unit in order to prepare assignments for the nursing staff. For which clients would augmentation of labor with oxytocin be contraindicated, or used cautiously? Select all that apply. an 18-year-old primipara client who is experiencing acute pain and refusing an epidural catheter a 27-year-old primipara in active labor for the past 4 hours a 33-year-old female who is 32 weeks' gestation in labor with twins a 44-year-old primipara diagnosed with gestational diabetes in active labor for the past 6 hours a 30-year-old multipara woman who has experienced premature rupture of membranes 5 days ago but is just now reporting it to the health care provider

a 44-year-old primipara diagnosed with gestational diabetes in active labor for the past 6 hours a 33-year-old female who is 32 weeks' gestation in labor with twins a 30-year-old multipara woman who has experienced premature rupture of membranes 5 days ago but is just now reporting it to the health care provider

When caring for a client during the active phase of labor without continuous electronic fetal monitoring, the nurse would intermittently assess FHR every a. 15 to 30 mins b. 5 to 10 mins c. 45 to 60 mins d. 60 to 75 mins

a. 15 to 30 mins

When determining the frequency of contractions the nurse would measure which period of time? a. start of one contraction to the start of the next contraction b. beginning of one contraction to the end of the same contraction c. peak of one contraction to the peak of next contraction d. end of one contraction to the beginning of the next contraction

a. Start of one contraction to the start of the next contraction

Which practice would not be included in a physiologic birth? a. early induction of labor before 39 weeks gestation b. freedom of movement for the laboring woman c. continuous presence and support throughout labor d. encouraging spontaneous pushing when urge is felt

a. early induction of labor before 39 weeks gestation

Which assessment finding indicates positive bonding between the parents and tehir newborn? a. holding the infant close to the body b. having visitors hold the infant c. buying expensive infant clothes d. requesting that the nurses care for the infant

a. holding the infant close to the body

The major purpose of the first postpartum home care visit is to a. identify complications that require interventions b. obtain a blood specimen for PKU testing c. complete the official birth certificate d. support the new parents in their parenting roles

a. identify complications that require interventions

Which interventions are underutilized in promoting a normal birth? Select all that apply. a. oral nutrition and fluids in labor b. open-glottis pushing in the second stage of labor c. skin to skin contact after birth d. routine artificial rupture of membranes (amniotomy) e. labor induction with intravenous Pitocin f. routine episiotomy to shorten labor length

a. oral nutrition and fluids in labor b. open-glottis pushing in the second stage of labor c. skin to skin contact after birth

A nurse is assessing a client during the postpartum period. Which findings indicate normal postpartum adjustment? Select all that apply. passing gas tender abdomen nondistended abdomen abdominal pain active bowel sounds

active bowel sounds passing gas nondistended abdomen

A multigravid client has been in labor for several hours and is becoming anxious and distressed with the intensity of the frequent contractions. The nurse observes moderate bloody show and performs a vaginal examination to assess the progress of labor. The cervix is 9 cm dilated. The nurse knows that the client is in which phase of labor? perineal phase pelvic phase active phase latent phase

active phase

All pain management modalities can slow labor if given too early except: opioids acupuncture hydrotherapy epidural anesthesia

acupuncture

Which intervention would be helpful to a bottle-feeding client who's experiencing hard or engorged breasts? applying warm compresses administering bromocriptine restricting fluids applying ice

applying ice

Seven hours ago, a multigravida woman gave birth to a male infant weighing 4,133 g. She has voided once and calls for a nurse to check because she states that she feels "really wet" now. Upon examination, her perineal pad is saturated. The immediate nursing action is to: call the primary care provider or the nurse-midwife. increase the flow of an IV. inspect the perineum for lacerations. assess and massage the fundus.

assess and massage the fundus.

A pregnant woman is admitted to the hospital with a diagnosis of placenta previa. Which action would be the priority for this woman on admission? helping the woman remain ambulatory to reduce bleeding assessing fetal heart tones by use of an external monitor assessing uterine contractions by an internal pressure gauge performing a vaginal examination to assess the extent of bleeding

assessing fetal heart tones by use of an external monitor

A nurse is caring for a pregnant client with heart disease in a labor unit. Which intervention is most important in the first 48 hours postpartum? assessing for cardiac decompensation limiting sodium intake inspecting the extremities for edema ensuring that the client consumes a high fiber diet

assessing for cardiac decompensation

A laboring woman is admitted to the labor and birth suite at 4 ccm dilation. She would be in which phase of labor? a. latent b. active c. late d. early

b. active

Which suggestion would be most appropriate to include in the teaching plan for a postpartum woman who needs to lose weight? a. increase fluid intake and acid-producing foods in her diet b. avoid empty calorie foods, breastfeed, and increase exercise c. start a high-protein, low-carb diet, and restrict fluids d. eat no snacks or carbs after dinner

b. avoid empty calorie foods, breastfeed, and increase exercise

Which condition would most likely cause a pregnant woman with type 1 diabetes the greatest difficulty during her pregnancy? a. placenta previa b. hyperemesis gravidarum c. placental abruption d. Rh incompatibility

b. hyperemesis gravidarum

Which fetal lie is most conductive to a spontaneous vaginal birth? a. transverse b. longitudinal c. perpendicular d. oblique

b. longitudinal

As the nurse is explaining the difference between true versus false labor to her childbirth class, she states that the major difference between them is a. discomfort level is greater with false labor b. progressive cervical changes occur in true labor c. there is a feeling of nausea with false labor d. there is more fetal movement with true labor

b. progressive cervical changes occur in true labor

The nurse is caring for a woman experiencing hypertonic uterine dystocia. The woman's contractions are erratic in their frequency, duration, and of high intensity. The priority nursing intervention would be to: a. encourage ambulation every 30 minutes b. provide pain relief measures c. monitor the oxytocin infusion rate closely d. prepare the woman for an amniotomy

b. provide pain relief measures

Which information would the nurse include when teaching a pregnant woman about the pathophysiologic mechanisms associated with gestational diabetes? a. pregnancy fosters the development of carbohydrate cravings b. there is progressive resistance to the effects of insulin c. hypoinsulinemia develops early in the first trimester d. glucose levels decrease to accomodate fetal growth

b. there is progressive resistance to the effects of insulin

During the fourth stage of labor, the nurse assesses the woman at frequent intervals after giving childbirth. What assessment data would cause the nurse the most concern? a. moderate amount of dark red locia drainage on peripad b. uterine fundus palpated to the right of the umbilicus c. an oral temperature of 100.6 perineal area bruised and edematous beneath her ice pack

b. uterine fundus palpated to the right of the umbilicus

The nurse is instructing the postpartum client who plans to bottle feed her newborn about measures to prevent breast engorgement when she is discharged. Which measure should the nurse include in the teaching plan? a. decreasing her fluid intake for the first week at home b. wearing a tight-fitting support bra 24 hours daily c. take a diuretic to release the extra fluid in the breasts d. manually express the milk that is accumulating

b. wearing a tight-fitting support bra 24 hours daily

A pregnant client is being discharged from the labor and birth suite because of false labor. The client asks the nurse how to tell whether the contractions are true contractions or Braxton Hicks contractions. Which description(s) will the nurse mention as characteristic of true contractions? Select all that apply . begin irregularly but become regular and predictable begin and remain irregular often disappear with ambulation or sleep felt first in lower back and sweep around to the abdomen in a wave increase in duration, frequency, and intensity felt first abdominally and remain confined to the abdomen and groin

begin irregularly but become regular and predictable felt first in lower back and sweep around to the abdomen in a wave increase in duration, frequency, and intensity

A nurse is reading a journal article about the use of real-time ultrasonography, which allows the health care provider to obtain information about the fetus. The nurse would expect the article to describe which type of information? the effectiveness of neural tube defect treatment biophysical profile chromosomal abnormalities the size and shape of placenta

biophysical profile

A nurse working with a woman in preterm labor receives a telephone report for the fetal fibronectin test done 10 hours ago. The report indicates an absence of the protein, which the nurse knows indicates: no infection is present. birth is likely within the next 2 weeks. infection is present. birth is unlikely within the 2 next weeks.

birth is unlikely within the 2 next weeks.

A pregnant woman in her 39th week of pregnancy presents to the clinic with a vaginal infection. She tests positive for chlamydia. What would this disease make her infant at risk for? chickenpox deafness neonatal laryngeal papillomas blindness

blindness

A fetus is experiencing shoulder dystocia during birth. The nurse would place priority on performing which fetal assessment postbirth? extensive lacerations assess for cleft palate brachial plexus assessment monitor for a cardiac anomaly

brachial plexus assessment

A new young mother has tested positive for HIV. When discussing the situation with the client, the nurse should advise the mother that she should avoid which activity? future pregnancies handling the infant with open sores cesarean birth breastfeeding

breastfeeding

A new mother has been reluctant to hold her newborn. Which action by the nurse would help promote this mother's attachment to her newborn? showing a video of parents feeding their babies talking about how the nurse held her own newborn while on the birthing table bringing the newborn into the room allowing the mother to pick the best time to hold her newborn

bringing the newborn into the room

A nurse is providing care to a woman in labor. When reviewing the woman's medical record, the nurse notes that fetal position is documented as LSA. The nurse interprets this to mean that which part of the fetus is presenting? acromion process chin occiput buttocks

buttocks

The nurse notes the presence of transient fetal accelerations on the fetal monitoring strip. Which intervention would be most appropriate? a. reposition the client on the left side b. begin 100% oxygen via face mask 5. document this as indicating a normal pattern d. call the health care provider immediately

c. Document this normal pattern

In the taking-in maternal role phase described by Rubin, the nurse would expect the womans behavior to be characterized in what way? a. gaining self-confidence b. adjusting to her new relationships c. being passive and dependent d. resuming control over her life

c. being passive and dependent

After teaching a group of breastfeeding women about nutritional needs, the nurse determines that the teaching was successful when the women state that they need to increase their intake of which nutrients? a. carbs and fiber b. fats and vitamines c. calories and protein d. iron-rich foods and minerals

c. calories and protein

Women who drink alcohol during pregnancy a. often produce more alcohol dehydrogenase b. usually become intoxicated more quickly than before c. can give birth to an infant with fetal alcohol spectrum disorder d. gain fewer pounds throughout gestation

c. can give birth to an infant with fetal alcohol spectrum disorder

Which assessment would indicate that a woman is in true labor? a. membranes ruptured and fluid is clear b. presenting part is engaged and not floating c. cervix is 4 cm dilated, 90% effaced d. contractions last 30 seconds every 5 to 10 minutes

c. cervix is 4 cm dilated, 90% effaced

By the end of the second stage of laborn, the nurse would expect which of the following events? a. the cervix is fully dilated and effaced b. the placenta is detached and expelled c. the fetus is born and on mother's chest d. the woman may request pain medications

c. fetus is born and on mother's chest

Physiologic preparation for labor would be demonstrated by a. a decrease in braxton hicks contractions felt by mother b. weight gain and an increased appetite by mother c. lightening when the fetus drops into true pelvis d. fetal heart rate accelerations and increased movements

c. lightening when the fetus drops into true pelvis

When assessing a postpartum woman, which finidng would lead the nurse to suspect postpartum blues? a. panic attacks and suicidal thoughts b. anger toward self and infant c. periodic crying and insomnia d. obsessive thoughts and hallucinations

c. periodic crying and insomnia

A client in labor is agitated and nervous about the birth of her child. The nurse explains to the client that fear and anxiety cause the release of certain compounds that can prolong labor. The nurse is referring to which compounds? prostaglandins oxytocin relaxin catecholamines

catecholamines

In preparing a class for a group of pregnant couples, the nurse includes information about possible newborn complications associated with smoking during pregnancy. Which complications will the nurse include? Select all that apply. sudden infant death syndrome low birth weight cleft lip and palate trisomy 21 cerebral palsy

cerebral palsy low birth weight cleft lip and palate sudden infant death syndrome

A nurse is teaching a group of pregnant women about the signs that labor is approaching. When describing these signs, which sign would the nurse explain as being essential for effacement and dilation (dilatation) to occur? Braxton Hicks contractions lightening cervical ripening and softening bloody show

cervical ripening and softening

A nurse is teaching a group of pregnant women about the signs that labor is approaching. When describing these signs, which sign would the nurse explain as being essential for effacement and dilation (dilatation) to occur? bloody show cervical ripening and softening lightening Braxton Hicks contractions

cervical ripening and softening

When explaining to a class of pregnant women why labor begins, the nurse will include the fact that there are several theories that have been proposed to explain why labor begins, although none have been proven scientifically. Which idea is one of those theories? change in estrogen-to-progesterone ratio decrease in prostaglandins, leading to myometrium contractions decrease in the level of estrogen decrease in number of oxytocin receptors

change in estrogen-to-progesterone ratio

A woman with cardiac disease has come to the office for prenatal counseling. Assessment supports the decision to caution the woman against pregnancy. The woman most likely fits the criteria for which functional risk classification? class IV class II class III class I

class IV

When providing nutritional counseling to a pregnant woman with diabetes, the nurse would urge the client to obtain most of her calories from which source? protein saturated fats complex carbohydrates unsaturated fats

complex carbohydrates

Which intervention would be most important when caring for the client with breech presentation confirmed by ultrasound? applying suprapubic pressure against the fetal back continuing to monitor maternal and fetal status noting the space at the maternal umbilicus auscultating the fetal heart rate at the level of the umbilicus

continuing to monitor maternal and fetal status

Which finding would lead the nurse to suspect that the fetus of a woman in labor is in hypertonic uterine dysfunction? fetal buttocks as the presenting part reports of severe back pain contractions most forceful in the middle of uterus rather than the fundus lack of cervical dilation past 2 cm

contractions most forceful in the middle of uterus rather than the fundus

After the nurse provides instructions to a postpartum woman about postpartum blues, which statement indicates understanding? a. I will need to take medication daily to treat the anxiety and sadness b. I will call the OB support line only if I start to hear voices b. I will contact my doctor if I become dizzy and feel nauseated d. I will feel like laughing one minute and crying the next minute

d. I will feel like laughing one minute and crying the next minute

When assessing the following women, who would the nurse identify as being at the greatest risk for preterm labor? a. woman who had twins in previous pregnancy b. client living in a large city close to the subway c. woman working full-time as a computer programmer d. client with a history of a previous preterm birth

d. client with a history of a previous preterm birth

The nurse is explaining to a postpartum woman 48 hours after childbirth that the afterpains she is experiencing can be the result of which factor? a. abdominal cramping as a sign of endometriosis b. a small infant weighing less than 8 lb c. pregnancies that were too closely spaced d. contractions of the uterus after birth

d. contractions of the uterus after birth

A new mother gave birth 12 hours ago. Because this is her first child, which goal planned by the nurse is most appropriate? a. early discharge for the mother and newborn b. rapid transition into the role of being a parent/caregiver c. minimal need for expression of feelings now d. effective education of both parents before discharge

d. effective education of both parents before discharge

Women who are obese have a greater risk of developing which condition during pregnancy? a. T1DM b. Hypotension c. Low-birth-weight infant d. gestational hypertension

d. gestational hypertension

The rationale for using a prostaglandin gel for a client prior to the induction of labor is to: a. stimulate uterine contractions b. numb cervical pain receptors c. prevent cervical lacerations d. soften and efface the cervix

d. soften and efface the cervix

A woman received morphine during labor to help with pain control. Which finding would the nurse need to monitor the newborn for after birth? decreased alertness increased crying low Apgar increased agitation

decreased alertness

A client in the active phase of labor is diagnosed as having a protracted labor pattern. Which pattern would the nurse assess as indicative of a protracted labor pattern? delayed descent of the fetal head arrest of the descent of the fetal head prolonged deceleration phase secondary arrest of cervical dilation (dilatation)

delayed descent of the fetal head

The student nurse is learning about normal labor. The teacher reviews the cardinal movements of labor and determines the instruction has been effective when the student correctly states the order of the cardinal movements as follows: internal rotation, descent, extension, flexion, external rotation, expulsion descent, flexion, internal rotation, extension, external rotation, expulsion internal rotation, flexion, descent, extension, external rotation, expulsion descent, flexion, external rotation, extension, internal rotation, expulsion

descent, flexion, internal rotation, extension, external rotation, expulsion

The nurse explains Leopold maneuvers to a pregnant client. For which purposes are these maneuvers performed? Select all that apply. determining the size of the fetus determining the position of the fetus determining the lie of the fetus determining the weight of the fetus determining the presentation of the fetus

determining the presentation of the fetus determining the position of the fetus determining the lie of the fetus

A nurse is describing to a group of young parents the many changes that will occur during the early postpartum period. The nurse reviews common reports experienced as the woman's body returns to her prepregnancy state. The nurse determines that the teaching was successful when the participants identify which report as being most common during the first week (indicating that fluid volume is returning to normal)? nocturia urinary frequency diaphoresis urinary urgency

diaphoresis

The nurse, assessing the lochia of a client, attempts to separate a clot and identifies the presence of tissue. Which observation would indicate the presence of tissue? easy to separate clots difficult to separate clots foul-smelling lochia yellowish-white lochia

difficult to separate clots

A woman is being closely monitored and treated for severe preeclampsia with magnesium sulfate. Which finding would alert the nurse to the development of magnesium toxicity in this client? seizures serum magnesium level of 6.5 mEq/L elevated liver enzymes diminished reflexes

diminished reflexes

A nurse is caring for a woman in labor and understands that as the fetus travels through the birth canal, the fetus makes positional changes that occur concurrently. Based on the nurse's conceptualization of their sequential occurrence, list the cardinal movements of labor in the correct order that the nurse would expect the fetus to move. All options must be used. engagement extension expulsion flexion internal rotation

engagement flexion internal rotation extension expulsion

The nurse is assessing the laboring client to determine fetal oxygenation status. What indirect assessment method will the nurse likely use? fetal position external electronic fetal monitoring fetal oxygen saturation fetal blood pH

external electronic fetal monitoring

A woman who immigrated here from a third world country presents to the clinic to find out if she is pregnant. Which signs and/or symptoms would the nurse assess as possible indicators that she might have an active case of tuberculosis as well? Select all that apply. weight gain anorexia fatigue hemoptysis night sweats

fatigue night sweats hemoptysis anorexia

A client at 35 weeks' gestation is now in stable condition after being admitted for vaginal bleeding. Which assessment should the nurse prioritize? fetal heart tones signs of shock infection uterine stabilization

fetal heart tones

A nurse is conducting a class for a group of pregnant women who are near term. As part of the class, the nurse is describing the process of attachment and bonding with their soon to be newborn. The nurse determines that the teaching was successful when the group states that bonding typically develops during which time frame after birth? first 6 months first 30 to 60 minutes first month first 3 to 5 days

first 30 to 60 minutes

The nurse is preparing a postpartum nursing care plan for a single HIV-positive primigravida client. The nurse should prioritize in the plan how to acquire which resource? breast pump diapers car seat formula

formula

A nurse is caring for a client who has just received an episiotomy. The nurse observes that the laceration extends through the perineal area and continues through the anterior rectal wall. How does the nurse classify the laceration? second degree fourth degree third degree first degree

fourth degree

A nurse is caring for a client in her third stage of labor. The nurse would predict the placenta is separating from the uterus based on which assessment findings? Select all that apply. fresh gushing of blood from the vagina falling downward of uterus in the abdomen umbilical cord descending lower down a globular shaped uterus a relaxed and distended uterus

fresh gushing of blood from the vagina umbilical cord descending lower down a globular shaped uterus

A pregnant client has been diagnosed with gestational diabetes. Which are risk factors for developing gestational diabetes? Select all that apply. previous large-for-gestational-age (LGA) infant hypertension obesity maternal age less than 18 years genitourinary tract abnormalities

genitourinary tract abnormalities obesity hypertension previous large-for-gestational-age (LGA) infant

The nurse is caring for a client in active labor. Which assessment finding requires health care provider notification? nausea hyperventilation gross proteinuria elevated WBC count

gross proteinuria

A client in active labor is given spinal anesthesia. Which information would the nurse include when discussing with the client and family about the disadvantages of spinal anesthesia? headache following anesthesia passage of the drug to the fetus increased frequency of micturition excessive contractions of the uterus

headache following anesthesia

A primigravida at 28 weeks' gestation comes to the clinic for a checkup. She tells the nurse that her mother gave birth to both of her children prematurely, and she is afraid that the same will happen to her. Which risk factors associated with preterm birth would the nurse discuss with the client? Select all that apply. current multiple gestation pregnancy previous cesarean birth large-for-gestational-age fetus history of previous preterm birth uterine or cervical abnormalities

history of previous preterm birth current multiple gestation pregnancy uterine or cervical abnormalities

Which changes in pregnancy would the nurse identify as a contributing factor for arterial thrombosis, especially for the woman with atrial fibrillation? hypercoagulable state increase in blood volume increased cardiac output elevation of diaphragm

hypercoagulable state

A client has been in labor for 10 hours, with contractions occurring consistently about 5 minutes apart. The resting tone of the uterus remains at about 9 mm Hg, and the strength of the contractions averages 21 mm Hg. The nurse recognizes which condition in this client? uncoordinated contractions hypertonic contractions Braxton Hicks contractions hypotonic contractions

hypotonic contractions

A client in her sixth week postpartum reports general weakness. The client has stopped taking iron supplements that were prescribed to her during pregnancy. The nurse would assess the client for which condition? hyperglycemia hypertension hypovolemia hypothyroidism

hypovolemia

A woman who is breastfeeding her newborn reports that her breasts seem quite full. Assessment reveals that her breasts are engorged. Which factor would the nurse identify as the most likely cause for this development? inability of infant to empty breasts cracking of the nipple inadequate secretion of prolactin improper positioning of infant

inability of infant to empty breasts

A nurse is caring for a pregnant client who is in labor. Which maternal physiologic responses should the nurse monitor for in the client as the client progresses through birth? Select all that apply. increase in respiratory rate increase in gastric emptying and pH increase in blood pressure increase in heart rate slight decrease in body temperature

increase in heart rate increase in blood pressure increase in respiratory rate

The nurse is reviewing the laboratory test results of a client in labor. Which finding would the nurse consider normal? decreased plasma fibrinogen levels increased white blood cell count increased blood coagulation time increased blood glucose levels

increased white blood cell count

A nursing instructor is teaching students about fetal presentations during birth. The most common cause for increased incidence of shoulder dystocia is: increasing birth weight. longer length of labor. increased number of overall pregnancies. poor quality of prenatal care.

increasing birth weight.

A client appears to be resting comfortably 12 hours after giving birth to her first child. In contrast, she labored for more than 24 hours, the primary care provider had to use forceps to deliver the baby, and she had multiple vaginal examinations during labor. Based on this information what postpartum complication is the client at risk for developing? pulmonary emboli depression infection hemorrhage

infection

A pregnant woman with chronic hypertension comes to the clinic for evaluation. The last several blood pressure readings have been gradually increasing. On today's visit her blood pressure is 166/100 mm Hg. The health care provider prescribes an antihypertensive agent. The nurse anticipates which agent as likely to be prescribed? labetalol albuterol hydroxychloroquine ipratropium

labetalol

The student nurse is preparing to assess the fetal heart rate (FHR) and has determined that the fetal back is located toward the client's left side, the small parts toward the right side, and there is a vertex (occiput) presentation. The nurse should initially begin auscultation of the fetal heart rate in the mother's: left upper quadrant. right lower quadrant. right upper quadrant. left lower quadrant.

left lower quadrant.

A nurse is meeting with a group of pregnant clients who are in their last trimester to teach them the signs that may indicate they are going into labor. The nurse determines the session is successful after the clients correctly choose which signs as an indication of starting labor? Select all that apply. lightening weight gain constipation bloody show backache

lightening bloody show backache

A nurse is assessing a breastfeeding client in the third week postpartum. During the physical examination, the nurse observes that the rugae in the vagina have not reappeared. Which factor would the nurse identify as the possible cause of delayed return of rugae? high circulating prolactin level low circulating progesterone level low circulating oxytocin level low circulating estrogen level

low circulating estrogen level

A client in labor has administered an epidural anesthesia. Which assessment findings should the nurse prioritize? maternal hypotension and fetal bradycardia maternal hypotension and fetal tachycardia maternal hypertension and fetal bradycardia maternal hypertension and fetal tachycardia

maternal hypotension and fetal bradycardia

The nurse is concerned that a client is not obtaining enough folic acid. Which test would the nurse anticipate being used to evaluate the fetus for potential neural tube defects? Doppler flow study triple-marker screen maternal serum alpha-fetoprotein analysis amniocentesis

maternal serum alpha-fetoprotein analysis

The nurse is assessing a client in labor for pain and notes she is currently not doing well handling the increased pain. Which opioid can the nurse offer to the client to assist with pain control? meperidine thiopental secobarbital hydroxyzine hydrochloride

meperidine

A pregnant client has tested positive for cytomegalovirus. What can this cause in the newborn? bicuspid valve stenosis microcephaly hypertension clubbed fingers and toes

microcephaly

A new mother talking to a friend states, "I wish my baby was more like yours. You are so lucky. My baby has not slept straight through the night even once. It seems like all she wants to do is breastfeed. I am so tired of her." This is an example of which behavior? positive attachment negative bonding positive bonding negative attachment

negative attachment

A 37-year-old primigravida client at 40 weeks' gestation is seen in the clinic for a scheduled prenatal visit. What report by the client would lead the nurse to predict the woman is close to labor? dilation (dilatation) nesting ripening of the cervix effacement

nesting

A pregnant client diagnosed with hyperemesis gravidarum is prescribed intravenous fluids for rehydration. When preparing to administer this therapy, which solution would the nurse anticipate being prescribed initially? normal saline albumin 0.45% sodium chloride dextrose 5% and water

normal saline

A pregnant client with preeclampsia with severe features has developed HELLP syndrome. In addition to the observations necessary for preeclampsia, what other nursing intervention is critical for this client? monitoring for infection administration of a tocolytic, if prescribed observation for bleeding maintaining a patent airway

observation for bleeding

Assessment reveals that the fetus of a client in labor is in the vertex presentation. The nurse determines that which part is presenting? occiput shoulders brow buttocks

occiput

The nurse is observing a client who gave birth yesterday. Where should the nurse expect to find the top of the client's fundus? at the level of the umbilicus one fingerbreadth above the umbilicus one fingerbreadth below the umbilicus below the symphysis pubis

one fingerbreadth below the umbilicus

A pregnant client is reporting of a large amount of malodorous vaginal discharge that is foamy and yellow-green in color, vaginal itching and painful intercourse. When asked, she also reports that urination is somewhat painful. She is diagnosed with trichomoniasis. What treatment would the nurse anticipate the client receiving? oral metronidazole benzathine penicillin G IM ceftriaxone IM oral erythromycin

oral metronidazole

A nurse is caring for a client who is nursing her baby boy. The client reports afterpains. Secretion of which substance would the nurse identify as the cause of afterpains? estrogen oxytocin prolactin progesterone

oxytocin

A woman who is pregnant for the first time has arrived to the labor department thinking she was in labor only to be diagnosed with Braxton Hicks contractions and sent home. Prior to leaving the unit, the woman asks, "How will I know when it is 'true' labor?" Which signs/symptoms should the woman associate with true labor? Select all that apply. pink-tinged blood and mucus mixture on underwear pain in back that wraps across the abdomen and increases in frequency and intensity leakage of white to yellow discharge from the nipples sudden gush of clear fluid coming from the vagina urine leakage after coughing or sneezing

pain in back that wraps across the abdomen and increases in frequency and intensity pink-tinged blood and mucus mixture on underwear sudden gush of clear fluid coming from the vagina

A client in labor has been admitted to the labor and birth suite. The nurse assessing the woman notes that the fetus is in a cephalic presentation. Which description should the nurse identify by the term presentation? relation of the different fetal body parts to one another relation of the fetal presenting part to the maternal ischial spine relationship of the presenting part to the maternal pelvis part of the fetal body entering the maternal pelvis first

part of the fetal body entering the maternal pelvis first

During the birth, the primary care provider performed an episiotomy. The client is now reporting discomfort. To reduce this discomfort and increase hygiene to the perineum, the nurse would encourage the client to use which intervention? baby wipes moist cloths peribottle and warm water alcohol wipes

peribottle and warm water

The nurse is assessing a woman with class III heart disease who is in for a prenatal visit. What would be the first recognizable sign that this client is in heart failure? low blood pressure persistent rales in the bases of the lungs audible wheezes elevated blood pressure

persistent rales in the bases of the lungs

When educating a group of nursing students about the different types of pelves, the nurse describes one type as being flat, having a wider transverse diameter than anterior-posterior diameter, with ischial spines that are wide apart, and a short sacrum. The students are correct when they identify this description with which type? gynecoid anthropoid android platypelloid

platypelloid

The nurse is assessing a client at a postpartum visit and notes the client is emotionally sensitive, complains about being a failure, and appears extremely sad. The nurse concludes the client is presenting with which potential condition? postpartum depression postpartum blues anxiety disorders postpartum psychosis

postpartum depression

At the 6-week visit following delivery of her infant, a postpartum client reports extreme fatigue, feelings of sadness and anxiety, and insomnia. Based on these assessment findings, the nurse documents that the client is exhibiting characteristics of: postpartum blues. postpartum adjustment. postpartum depression. postpartum psychosis.

postpartum depression.

A nurse is caring for a client on the second day postpartum. The client informs the nurse that she is voiding a large volume of urine frequently. Which factor should the nurse identify as a potential cause for urinary frequency? urinary tract infection urinary overflow postpartum diuresis trauma to pelvic muscles

postpartum diuresis

During a routine prenatal visit, a client is found to have 1+ proteinuria and a blood pressure rise to 140/90 mm Hg with mild facial edema. The nurse recognizes that the client has which condition? eclampsia preeclampsia without severe features gestational hypertension preeclampsia with severe features

preeclampsia without severe features

The nurse performs a nonstress test (NST) on a client at 36 weeks' gestation. What criteria on the tracing does the nurse use to determine that the NST is reactive? presence of 2 accelerations per hour presence of 2 decelerations in 60 minutes presence of 2 contractions during within 20 minutes presence of 2 accelerations in 20 minutes

presence of 2 accelerations in 20 minutes

The nurse is admitting a G3 P2 client at 38 weeks' gestation who arrived reporting painless bleeding from the vagina leading to the diagnosis of placenta previa. When questioned by the client as to what caused this, which most likely factor should the nurse point out in her answer? living in coastal areas previous cesarean birth maternal age more than 30 years morbidly obese

previous cesarean birth

When assessing fetal heart rate patterns, which finding would alert the nurse to a possible problem? accelerations early decelerations variable decelerations prolonged decelerations

prolonged decelerations

A woman is experiencing dystocia that appears related to psyche problems. Which intervention would be most appropriate for the nurse to initiate? administering oxytocin providing a comfortable environment with dim lighting encouraging the women to change positions frequently preparing the woman for an amniotomy

providing a comfortable environment with dim lighting

After assessing a client's progress of labor, the nurse suspects the fetus is in a persistent occiput posterior position. Which finding would lead the nurse to suspect this condition? reports of severe back pain contractions most forceful in the middle of uterus rather than the fundus lack of cervical dilation (dilatation) past 2 cm fetal buttocks as the presenting part

reports of severe back pain

A nurse is monitoring a female client with an epidural block. Which complication would be the most important for the nurse to monitor in the client? respiratory depression a failed block accidental intrathecal block postdural puncture (spinal) headache

respiratory depression

During the assessment of a laboring client, the nurse learns that the client has cardiovascular disease (CVD). Which assessment would be priority for the newborn? temperature respiratory function heart rate urine output

respiratory function

As part of a 31-year-old client's prenatal care, the nurse is assessing immunization history. Which immunization is most relevant to ensuring a healthy fetus? rubella hepatitis A and B measles diphtheria, tetanus, and pertussis

rubella

The nurse is working triage in the emergency department. The nurse realizes the client is in true labor when she states that the she is experiencing: expelling mucus plug. fetal engagement. ruptured membranes. contractions.

ruptured membranes.

A nurse is providing care to a pregnant client hospitalized with preeclampsia. The nurse immediately notifies the health care provider that the client has developed eclampsia based on which finding? seizure activity proteinuria hyperreflexia blood pressure greater than 160/100 mm Hg

seizure activity

When dealing with a pregnant adolescent, the nurse assists the client to integrate the tasks of pregnancy while at the same time fostering development of which trait? autonomy dependence trust self-identity

self-identity

Assessment of a woman in labor reveals that the scapula of the fetus is the presenting part. The nurse interprets this finding as indicating which fetal presentation? breech cephalic shoulder vertex

shoulder

A client gave birth 1 day ago and the nurse is monitoring the client's blood pressure. In which position will the nurse place the client to get the most accurate reading? lying on the right side for 5 minutes standing next to the bed after 3 minutes sitting on the side of the bed for 2 minutes lying flat in the bed on the back

sitting on the side of the bed for 2 minutes

A nurse is caring for a newborn with fetal alcohol spectrum disorder. What characteristic of the fetal alcohol spectrum disorder should the nurse assess for in the newborn? poor breathing pattern small head circumference decreased blood glucose level wide eyes

small head circumference

Which of these cardiac variations, if found in the client who is pregnant, should the nurse recognize as a normal finding in pregnancy? split S1S2 soft systolic murmur S4 (atrial gallop) premature ventricular contractions

soft systolic murmur

A pregnant client requires administration of an epidural block for management of pain during labor. For which conditions should the nurse check the client before administering the epidural block? Select all that apply. skin rashes or bruises coagulation defects hypovolemia varicose veins spinal abnormality

spinal abnormality hypovolemia coagulation defects

Over the past 20 weeks, the following blood pressure readings are documented for a pregnant client with chronic hypertension: week 16 - 130/86 mm Hg; week 20 - 138/88 mm Hg; week 24 - 136/82 mm Hg; and week 28 - 138/88 mm Hg. The nurse interprets these findings as indicating which classification of her blood pressure? elevated stage 1 hypertensive crisis stage 2

stage 1

What is the role of the nurse during the preconception counseling of a pregnant client with chronic hypertension? stressing the increased use of Vitamin D supplements stressing the positive benefits of a healthy lifestyle stressing regular walks and exercise stressing the avoidance of dairy products

stressing the positive benefits of a healthy lifestyle

Which factor might result in a decreased supply of breast milk in a postpartum client? maternal diet high in vitamin C frequent feedings supplemental feedings with formula an alcoholic drink

supplemental feedings with formula

An episiotomy or a cesarean incision requires assessment. Which assessment criterion for skin integrity is not initially noted? edema redness drainage temperature

temperature

The nursing student demonstrates an understanding of dystocia with which statement? "Dystocia is not diagnosed until after the birth." "Dystocia is diagnosed at the start of labor." "Dystocia cannot be diagnosed until just before birth." "Dystocia is diagnosed after labor has progressed for a time."

"Dystocia is diagnosed after labor has progressed for a time."

When assessing a new father's adaptation to his new role, which statement would indicate that he is in the reality stage? "It'll be fun to have a baby in the house, but things shouldn't change too much." "I may not be a pro at helping out with the baby, but I enjoy being involved." "I've learned how to diaper and bathe the baby so I can be a really involved dad." "I didn't realize all that went into being a dad. I wasn't prepared for this."

"I didn't realize all that went into being a dad. I wasn't prepared for this."

A nurse is teaching a woman diagnosed with gestational diabetes about meal planning and nutrition. The nurse determines that additional teaching is needed based on which client statement? "I should get most of my calories from good complex carbs." "I need to avoid any fat with my meals." "It's okay to eat small meals or snacks throughout the day." "Having a bedtime snack is good for me."

"I need to avoid any fat with my meals."

A mother is talking to the nurse and is concerned about managing her asthma while she is pregnant. Which response to the nurse's teaching indicates that the woman needs further instruction? "It is fine for me to use my albuterol inhaler if I begin to feel tight." "I will monitor my peak expiratory flow rate regularly to help me predict when an asthma attack is coming on." "I need to begin taking allergy shots like my friend to prevent me from having an allergic reaction this spring." "I need to be aware of my triggers and avoid them as much as possible."

"I need to begin taking allergy shots like my friend to prevent me from having an allergic reaction this spring."

A client who gave birth vaginally 16 hours ago states she does not need to void at this time. The nurse reviews the documentation and finds that the client has not voided for 7 hours. Which response by the nurse is indicated? "If you don't attempt to void, I'll need to catheterize you." "I'll contact your primary care provider." "It's not uncommon after birth for you to have a full bladder even though you can't sense the fullness." "I'll check on you in a few hours."

"It's not uncommon after birth for you to have a full bladder even though you can't sense the fullness."

The nurse is teaching a prenatal class on potential problems during pregnancy to a group of expectant parents. The risk factors for placental abruption (abruptio placentae) are discussed. Which comment validates accurate learning by the parents? "If I develop this complication, I will have bright red vaginal bleeding," "Placental abruption is quite painful and I will need to let the doctor know if I begin to have abdominal pain." "I need a cesarean section if I develop this problem." "Since I am over 30, I run a much higher risk of developing this problem."

"Placental abruption is quite painful and I will need to let the doctor know if I begin to have abdominal pain."

The client is anxious about her prolonged pregnancy. She informs the nurse she has been doing research on the Internet and has read about certain herbs that can help to induce labor. Which response from the nurse would be appropriate? "There is no scientific evidence they work. You will just complicate your situation more." "Why would you do something as stupid as that?" "Please talk to your primary care provider first to ensure it is safe." "Personally, I would use them, but I cannot tell you to."

"Please talk to your primary care provider first to ensure it is safe."

A woman's baby is HIV positive at birth. She asks the nurse if this means the baby will develop AIDS. Which statement would be the nurse's best answer? "The antibodies may be those transferred across the placenta; the baby may not develop AIDS." "HIV is transmitted at birth; having a cesarean birth prevented transmission." "She already has AIDS. That's what being HIV positive means." "HIV antibodies do not cross the placenta; this means the baby will develop AIDS."

"The antibodies may be those transferred across the placenta; the baby may not develop AIDS."

A client who had a vaginal birth 2 days ago asks the nurse when she will be able to breathe normally again. Which response by the nurse is accurate? "It usually takes about 3 months before all of your abdominal organs return to normal, allowing you to breathe normally." "Within 1 to 3 weeks, your diaphragm should return to normal, and your breathing will feel like it did before your pregnancy." "Everyone is different, so it is difficult to say when your respirations will be back to normal." "You should notice a change in your respiratory status within the next 24 hours."

"Within 1 to 3 weeks, your diaphragm should return to normal, and your breathing will feel like it did before your pregnancy."

A nurse is performing a vaginal examination of a woman in the early stages of labor. The woman has been at 2 cm dilated for the past 2 hours, but effacement has progressed steadily. Which statement by the nurse would best encourage the client regarding her progress? Don't mention anything to the client yet; wait for further dilation (dilatation) to occur. "You are still 2 cm dilated, but the cervix is thinning out nicely." "There has been no further dilation (dilatation); effacement is progressing." "You haven't dilated any further, but hang in there; it will happen eventually."

"You are still 2 cm dilated, but the cervix is thinning out nicely."

When a client is counseled about the advantages of epidural anesthesia, which statement made by the counselor would indicate the need for further teaching? "Epidural anesthesia is more effective than opioid analgesia in providing pain relief." "You can continuously receive epidural anesthesia until you have the baby, and even afterward if you need it." "You have no trouble walking around and using the bathroom after you receive the epidural." "If you end up having a cesarean, the epidural can be used for anesthesia during surgery."

"You have no trouble walking around and using the bathroom after you receive the epidural."

A client with asthma is confused by her primary care provider continuing her medication while she is pregnant, since she read online the medications can cause birth defects. What is the nurse's best response? "They won't cause any major defects." "I'll let your primary care provider know how you feel about it." "Your primary care provider will order safe doses of your medication." "It's OK to not use them if you would feel more comfortable."

"Your primary care provider will order safe doses of your medication."

A pregnant client presents for her first prenatal visit. She informs the nurse that she had an ectopic pregnancy 3 years ago. She ask the nurse if this would happen this time. Which response by the nurse would be best? "Be calm. Why worry about things that likely won't happen?" "Just because you had one ectopic pregnancy does not mean you will have another." "You should not worry about this right now—stress can harm the fetus." "Your statistical risk of another tubal pregnancy is increased."

"Your statistical risk of another tubal pregnancy is increased."

A client has just received combined spinal epidural. Which nursing assessment should be performed first? Assess for progress in labor. Assess pain level using a pain scale. Assess for spontaneous rupture of membranes. Assess for fetal tachycardia. Assess vital signs.

Assess vital signs.

After conducting a refresher class on possible congenital infections with a group of perinatal nurses, the nurse recognizes the class was successful when the group identifies which congenital viral infection as the most common? HIV CMV HPV RSV

CMV

An older female pregnant with her first child develops some pain in her legs associated with warmth to touch. Suspecting a blood clot, an ultrasound is prescribed and a peripheral venous thrombosis is diagnosed. Which intervention was likely prescribed for this woman? "Buy and wear medical support hose every day." "If you sit at a desk, set a timer and get up and walk every 2 to 3 hours." "Take your blood thinner the same time each day." "Take a baby aspirin every morning and evening to prevent further clot formation."

"Buy and wear medical support hose every day."

A client who is in her first trimester is anxious to have an ultrasound at each visit. The nurse explains that it is not necessary and schedules a second ultrasound to be performed when she is about: 21 to 23 weeks' pregnant. 15 to 17 weeks' pregnant. 24 to 26 weeks' pregnant. 18 to 20 weeks' pregnant.

18 to 20 weeks' pregnant.

The nurse determines a client is 7 cm dilated. What is the best response when asked by the client's partner how long the client will be in labor? "Your partner is still in early latent phase of labor; it is too early to estimate when they will give birth." "Your partner is in active labor; they are progressing at this point and we will keep you posted." "Your partner is in the active phase of labor, and birth will be within 2 to 3 hours, though it might be sooner." "Your partner is doing well and is in the second stage of labor; birth could be anytime now."

"Your partner is in active labor; they are progressing at this point and we will keep you posted."

Postpartum breast engorgement occurs 48 to 72 hours after giving birth. What physiological change influences breast engorgement? a. an increase in blood and lymph upply to the breasts b. an increase in estrogen and progesterone levels c. a dramatic increase in colostrum produc tion d. fluid retention in the breasts due to the intravenous fluids given during labor

a. an increase in blood and lymph upply to the breasts

As a woman enters the second stage of labor, which would the nurse expect to assess? falling asleep from exhaustion reports of feeling hungry and unsatisfied expressions of satisfaction with her labor progress feelings of being frightened by the change in contractions

feelings of being frightened by the change in contractions

A client is entering her 42nd week of gestation and is being prepared for induction of labor. The nurse recognizes that the fetus is at risk for which condition? dystocia macrosomia infection hemorrhage

macrosomia

A woman with no previous history of heart disease begins to have symptoms of myocardial failure a few weeks before the birth of her first child. Findings include shortness of breath, chest pain, and edema, with her heart also showing enlargement. The nurse suspects which condition? pulmonary valve stenosis left-sided heart failure mitral stenosis peripartum cardiomyopathy

peripartum cardiomyopathy

A nurse is caring for a client who has had a cesarean birth with general anesthesia. The nurse would assess the woman closely for which possible complication? pruritus inadequate pain block maternal hypotension uterine atony

uterine atony A complication of general anesthesia is the relaxation of the uterine muscles, leading to uterine atony and possible postpartum hemorrhage.

During a routine assessment the nurse notes the postpartum client is tachycardic. What is a possible cause of tachycardia? uterine atony delayed hemorrhage extreme diaphoresis bladder distention

delayed hemorrhage

A pregnant client's labor has been progressing slower than normal. The client is visibly anxious and tense, telling the nurse, "I am so worried about what is going to happen. And I am so tired and feel so helpless." Other underlying issues that may be contributing to the client's slow labor progress have been ruled out. Which response(s) by the nurse would be appropriate? Select all that apply. "I will have to stop giving you pain medicine because it is slowing your labor." "I will keep you updated often on how you and your baby are doing." "Things are moving along but sometimes it can take a little longer." "Let me leave you alone for a little while so you can get some rest." "Maybe dimming the lights or some soft music will help you relax a bit."

"Maybe dimming the lights or some soft music will help you relax a bit." "I will keep you updated often on how you and your baby are doing." "Things are moving along but sometimes it can take a little longer."

A woman birth her infant 24 hours ago by cesarean. Which assessment findings should be reported to the assigned nurse? Select all that apply. Uterus feels boggy. The client reports breakthrough pain level of 7 to 8. Bleeding is noted on the abdominal dressing 2 x 5 cm in size. The client's abdomen is mildly distended and bowel sounds are hypoactive. Fundal height is one fingerbreadth below the umbilicus.

Uterus feels boggy. The client reports breakthrough pain level of 7 to 8.

A postpartum woman has been unable to urinate since giving birth. When the nurse is assessing the woman, which finding would indicate that this client is experiencing bladder distention? Percussion reveals tympany. Lochia is less than usual. Bladder is nonpalpable. Uterus is boggy.

Uterus is boggy.

The nurse plays a major role in assessing the progress of labor. The nurse integrates understanding of the typical rule for monitoring labor progress. Which finding would the nurse correlate with this rule? fetus descends 2 cm per hour cervix dilates 2 cm per hour cervix dilates 1 cm per hour fetus descends 1 cm per hour

cervix dilates 1 cm per hour

The nurse is assessing a pregnant client at 37 weeks' gestation and notes the fetus is at 0 station. When questioned by the client as to what has happened, the nurse should point out which event has occurred? extension expulsion engagement flexion

engagement

It has been 8 hours since a woman gave birth vaginally to a healthy newborn. When assessing the woman's fundus, the nurse would expect to find it at: between the umbilicus and symphysis pubis. the level of the umbilicus. 2 cm below the umbilicus. 1 cm below the umbilicus.

the level of the umbilicus.

A pregnant woman diagnosed with cardiac disease 4 years ago is told that her pregnancy is a high-risk pregnancy. The nurse then explains that the danger occurs primarily because of the increase in circulatory volume. The nurse informs the client that the most dangerous time for her is when? in weeks 8 to 12 in weeks 12 to 20 in weeks 28 to 32 in weeks 20 to 28

in weeks 28 to 32

Which finding would lead the nurse to suspect that a postpartum woman was developing a complication? a. fatigue and irritability b. perineal discomfort and pink discharge c. pulse rate of 60 bpm d. swollen, tender, hot area on the breast

d. swollen, tender, hot area on the breast

Many clients experience a slight fever after birth especially during the first 24 hours. To what should the nurse attribute this elevated temperature? change in the temperature from the birth room infection dehydration fluid volume overload

dehydration

The nurse is completing the teaching for a newly pregnant client with a BMI of 23. Which statement by the client indicates an understanding of weight gain during this pregnancy? "I need to gain 25 to 35 pounds (11 to 16 kg) during this pregnancy." "I need to gain less than 25 pounds (11 kg) during this pregnancy." "I need to gain 1 pound (0.45 kg) per week throughout this pregnancy." "I need to gain 0.5 pounds (0.23 kg) per week during this pregnancy."

"I need to gain 25 to 35 pounds (11 to 16 kg) during this pregnancy."

The nurse is providing discharge education for a new mother regarding constipation. Which statement by the mother indicates that she understands what the nurse explained to her? "I will increase my intake of fruits and vegetables in my diet. I love to eat them anyhow." "I will avoid medications for constipation such as psyllium because it can upset the baby's stomach." "A good meal for me is cream of chicken soup, cheese toast, and ice cream for dessert." "It is all right to suppress the urge to have a stool for a few days to allow my stitches to heal."

"I will increase my intake of fruits and vegetables in my diet. I love to eat them anyhow."

A woman who delivered her infant 2 days ago asks the nurse why she wakes up at night drenched in sweat. She is concerned that this is a problem. The nurse's best reply would be: "Sweating is very normal for the first few days after childbirth because your body needs to get rid of all the excess water from pregnancy." "Many women sweat after delivery but you seem to be perspiring far more than normal. I'll call the doctor." "I need to get your vital signs and check your fundus to be sure you are not going into shock." "Often, when a postpartum woman perspires like you are reporting, it means that they have an infection."

"Sweating is very normal for the first few days after childbirth because your body needs to get rid of all the excess water from pregnancy."

The nurse is monitoring the client's vital signs and notes: 100.2oF (37.9oC), heart rate 82, respiratory rate 17, and blood pressure 124/78. The client has recently had an epidural. What is the best response when the client's partner asks if she is getting sick? "The fever may be due to the epidural." "We will continue to monitor the situation." "She's dehydrated and needs something to drink." "Have you been exposed to any illnesses recently?"

"The fever may be due to the epidural."

A woman who has been in labor for a few hours is now complaining of being hungry. Which response by the nurse would be best if the client asks for some food to eat? "You can have a protein supplement." "What would you like to eat?" "You could have some hard candy to suck on." "I can get you something soft and easy to digest, like pudding."

"You could have some hard candy to suck on."

The nurse is teaching a postpartum woman and her spouse about postpartum blues. The nurse would instruct the couple to seek further care if the client's symptoms persist beyond which time frame? 4 weeks 2 weeks 3 weeks 1 week

2 weeks

The health care provider has determined that the source of dystocia for a woman is related to the fetus size. The nurse understands that macrosomia would indicate the fetus would weigh: 2500 to 3000 g 3,500 g to 4000 g 4,000 g to 4500 g 3,000 g to 3500 g

4,000 g to 4500 g

A pregnant woman with type 2 diabetes is scheduled for a laboratory test of glycosylated hemoglobin (HbA1C). What does the nurse tell the client is a normal level for this test? 8% 12% 14% 6%

6%

A client with rheumatoid arthritis (RA) is in week 38 of her pregnancy. Which intervention should the nurse make with this client? Urge the client to be on bed rest. Ask the client to decrease her intake of salicylates. Perform the Snellen eye test. Advise the client to continue her normal dosage of methotrexate.

Ask the client to decrease her intake of salicylates.

The nurse is assessing a 35-year-old woman at 22 weeks' gestation who has had recent laboratory work. The nurse notes fasting blood glucose 146 mg/dl (8.10 mmol/L), hemoglobin 13 g/dl (130 g/L), and hematocrit 37% (0.37). Based on these results, which instruction should the nurse prioritize? Check blood sugar levels daily. Take daily iron supplements. Include iron-enriched foods in the diet. the signs and symptoms of urinary tract infection

Check blood sugar levels daily.

When a client in labor is fully dilated, which instruction would be most effective to assist her in encouraging effective pushing?] A. Hold your breath and push through entire contraction. B. Use chest-breathing with the contraction. C. Pant and blow during each contraction. D. Wait until you feel the urge to push.

D. Wait until you feel the urge to push.

A pregnant woman is being evaluated for HELLP. The nurse reviews the client's diagnostic test results. An elevation in which result would the nurse interpret as helping to confirm this diagnosis? white blood cells LDH platelet count hematocrit

LDH

Which information would the nurse emphasize in the teaching plan for a postpartum woman who is reluctant to begin taking warm sitz baths? Sitz baths increase the blood supply to the perineal area. Sitz baths cause perineal vasoconstriction and decreased bleeding. Sitz baths may lead to increased postpartum infection. The longer a sitz bath is continued, the more therapeutic it becomes.

Sitz baths increase the blood supply to the perineal area.

A primigravida client has come to the clinic for a prenatal checkup. What teaching topics would help to promote a healthy pregnancy for this client? More frequent tooth brushing is recommended to prevent caries related to ptyalism. Douching is recommended to decrease the risk of vaginal infections. Swimming in a pool is a recommended exercise during pregnancy. Applying lanolin ointment to the breasts is recommended to prevent cracked nipples.

Swimming in a pool is a recommended exercise during pregnancy.

A client has just given birth to a healthy baby boy, but the placenta has not yet delivered. What stage of labor does this scenario represent? First Third Second Fourth

Third

At what time is the laboring client encouraged to push? When the fetal head can be seen When the nurse wants the client to push When the cervix is fully dilated When the health care provider has arrived

When the cervix is fully dilated

After a vaginal examination, the nurse determines that the client's fetus is in an occiput posterior position. The nurse would anticipate that the client will have : a. Intense back pain b. Frequent leg cramps c. Nausea and vomiting d. A precipitous birth

a. intense back pain

What is the Bishop score used to assess? a. presence of bacterial vaginosis b. amount of amniotic fluid present c. overall fetal well-being in labor d. cervical readiness for induction

d. cervical readiness for induction

The nurse is making a home visit to a woman who is 5 days' postpartum. Which finding would concern the nurse and warrant further investigation? diaphoresis lochia rubra uterus 5 cm below umbilicus edematous vagina

lochia rubra

While caring for a client following a lengthy labor and birth, the nurse notes that the client repeatedly reviews her labor and birth and is very dependent on her family for care. The nurse is correct in identifying the client to be in which phase of maternal role adjustment? taking-hold letting-go taking-in acquaintance/attachment

taking-in

Rubin identified a series of changes that a new mother makes during the postpartum period. The correct sequence of these changes is: taking-in, holding-on, letting-go. taking-in, taking-hold, letting-go. taking-in, taking-on, letting-go. taking, holding-on, letting-go.

taking-in, taking-hold, letting-go.

A young client with a cardiac problem wants to get pregnant and tells the nurse that she is sad that she will never be able to have a baby. What is the best response by the nurse? "Cardiovascular problems are not a concern during pregnancy." "Because of improved management, more women with cardiac problems can complete pregnancies successfully." "Women with your problem should never get pregnant because the risks and dangers are too high for you and the fetus." "If you get pregnant, you are likely to face many complications."

"Because of improved management, more women with cardiac problems can complete pregnancies successfully."

A nurse is providing care to a postpartum woman who gave birth about 2 days ago. The client asks the nurse, "I haven't moved my bowels yet. Is this a problem?" Which response by the nurse would be most appropriate? "Let me call your health care provider about this problem." "I'll get a laxative prescribed so that you can move your bowels." "That's unusual. Are you making sure to eat enough?" "It might take up to a week for your bowels to return to their normal pattern."

"It might take up to a week for your bowels to return to their normal pattern."

A woman has been in labor for the past 8 hours, and she has progressed to the second stage of labor. However, after 2 hours with no further descent, the provider diagnoses "arrest of labor." The woman asks, "Why is this happening?" Which response is the best answer to this question? "Maybe your uterus is just tired and needs a rest." "More than likely you have cephalopelvic disproportion (CPD) where baby's head cannot make it through the canal." "Maybe your baby has developed hydrocephaly and the head is too swollen." "It is likely that your body has not secreted enough hormones to soften the ligaments so your pelvic bones can shift to allow birth of the baby."

"More than likely you have cephalopelvic disproportion (CPD) where baby's head cannot make it through the canal."

A woman refuses to have an epidural block because she does not want to have a postdural puncture (spinal) headache after birth. What would be the nurse's best response? "Your health care provider knows what is best for you." "Spinal headache is not a usual complication of epidural blocks." "The anesthesiologist will do her best to avoid this." "The pain relief offered will compensate for the discomfort afterward."

"This is meconium-stained fluid from the baby."

A client in her third trimester comes to the clinic for an evaluation. Assessment reveals that the cervix is thinning. The client says, "I know my cervix needs to dilate, but why does it get thinner?" Which response by the nurse would be appropriate? "It thins to let your baby change positions during labor." "Your cervix thins so that your contractions can increase." "Cervical thinning is a sign that you are in true labor." "You need the cervix to thin so it can stretch more easily."

"You need the cervix to thin so it can stretch more easily."

The nurse is providing care to a client in labor. On examination, the nurse determines the fetus is at -1 station. The nurse interprets this as indicating that the fetus is: 1 cm below the pubic bone. 1 cm above the ischial spines. 1 cm below the ischial spines. 1 cm above the pubic bone.

1 cm above the ischial spines.

A woman is to undergo labor induction. The nurse determines that the woman requires cervical ripening if her Bishop score is: 9. 5. 6. 7.

5.

Following the birth, the nurse is responsible for assessing the cord pH. The nurse recognizes that which value would be considered a normal pH? 6.8 7.4 7.0 7.2

7.2 The normal mean pH value range is 7.2 to 7.3.

A client gave birth to a child 3 hours ago and noticed a triangular-shaped gap in the bones at the back of the head of her newborn. The attending nurse informs the client that it is the posterior fontanel (fontanelle). The client is anxious to know when the posterior fontanel (fontanelle) will close. Which time span is the normal duration for the closure of the posterior fontanel (fontanelle)? 4 to 6 weeks 12 to 14 weeks 8 to 12 weeks 14 to 18 weeks

8 to 12 weeks

The nurse is caring for a client in the transition stage of labor. In which scenario would the nurse predict the use of forceps may be used to assist with the birth? Reduce risk of complications To lessen the mother's pain Abnormal position of the fetal head The fetus is descending too slowly

Abnormal position of the fetal head

A postpartum mother has the following lab data recorded: a negative rubella titer. What is the appropriate nursing intervention? Assess the rubella titer of the baby. Administer rubella vaccine before discharge. Notify the health care provider. No action needed.

Administer rubella vaccine before discharge.

A client in labor has requested the administration of opioids to reduce pain. At 2 cm cervical dilation (dilatation), she says that she is managing the pain well at this point but does not want it to get ahead of her. What should the nurse do? Advise the client to hold out a bit longer, if possible, before administration of the drug, to prevent slowing labor. Refuse to administer opioids because they can develop dependency in the client and the fetus. Agree with the client, and administer the drug immediately to keep the pain manageable. Explain to the client that opioids should only be administered an hour or less before birth.

Advise the client to hold out a bit longer, if possible, before administration of the drug, to prevent slowing labor.

A client is exhibiting signs of engorgement, but her milk is still flowing easily. Which suggestion should the nurse prioritize? Apply ice packs before a feeding. Wear a tight fitting bra at all times. Ensure the baby empties the breasts at each feeding Restrict fluid intake to 2 L each day.

Ensure the baby empties the breasts at each feeding

A client presents to the birthing center in labor. The client's membranes have just ruptured. Which assessment is the nurse's priority? FHR maternal comfort level fetal position signs of infection

FHR

The nurse is performing Leopold maneuvers as part of the initial assessment. Which action would the nurse do first? Determine flexion by pressing downward toward the symphysis pubis. Feel for the fetal buttocks or head while palpating the abdomen. Palpate for the presenting part in the area just above the symphysis pubis. Feel for the fetal back and limbs as the hands move laterally on the abdomen.

Feel for the fetal buttocks or head while palpating the abdomen.

A full-term pregnant client is being assessed for induction of labor. Her Bishop score is less than 6. Which prescription would the nurse anticipate? Insert a Foley catheter into the endocervical canal. Rupture membranes. Administer oxytocin intravenously at 10 mU/minute. Prepare the client for a cesarean birth.

Insert a Foley catheter into the endocervical canal.

The nurse is inspecting a new client's perineum. What action(s) will the nurse take for this client? Select all that apply. Inspect the episiotomy for sutures and to ensure that the edges are approximated. Gently palpate for any hematomas. Palpate the episiotomy for pain. Note any hemorrhoids. Place the client in Trendelenburg position for inspection.

Inspect the episiotomy for sutures and to ensure that the edges are approximated. Note any hemorrhoids. Gently palpate for any hematomas.

One hour after birth the nurse is assessing a neonate in the nursery. The nurse begins by assessing which parameters? Inspecting posture, color, and respiratory effort Checking for identifying birthmarks or skin injuries Determining chest and head circumference Auscultating bowel sounds, and measuring urine output

Inspecting posture, color, and respiratory effort

A woman receives magnesium sulfate as treatment for preterm labor. The nurse assess and maintains the infusion at the prescribed rate based on which finding? Absent deep tendon reflexes Urine output 22 ml/hour Respiratory rate of 16 breaths/minute Decreased fetal heart rate variability

Respiratory rate of 16 breaths/minute

A client who has given birth is being discharged from the health care facility. She wants to know how safe it would be for her to have intercourse. Which instructions should the nurse provide to the client regarding intercourse after birth? Use oral contraceptive pills (OCPs) for contraception. Avoid performing pelvic floor exercises. Avoid use of water-based gel lubricants. Resume intercourse if bright red bleeding stops.

Resume intercourse if bright red bleeding stops.

The nurse teaches a sedentary pregnant client with a BMI of 35 about the importance of healthy lifestyle during pregnancy. Which goal would be appropriate for this client? Participate in a daily aerobic dance program. Adhere to a weight reduction diet. Begin lifting weights for 30 minutes per day. Walk for 30 minutes 5 days a week.

Walk for 30 minutes 5 days a week.

A postpartum woman with an episiotomy asks the nurse about perineal care. Which recommendation would the nurse give? Wash her perineum with her daily shower. Refrain from washing lochia from the suture line. Avoid using soap for any perineal care. Use an alcohol wipe to wash her episiotomy line.

Wash her perineum with her daily shower.

The nurse caring for a client in preterm labor observes abnormal fetal heart rate (FHR) patterns. Which nursing intervention should the nurse perform next? fetal scalp stimulation administration of oxygen by mask application of vibroacoustic stimulation tactile stimulation

administration of oxygen by mask

After teaching a review class to a group of perinatal nurses about various methods for cervical ripening, the nurse determines that the teaching was successful when the group identifies which method as surgical? amniotomy breast stimulation laminaria prostaglandin

amniotomy

Immediately after childbirth in the recovery area, the nurse observes the mother's partner's fascination and interest in the new child. This behavior is often termed a. attachment b. engrossment c. bonding d. temperament

b. engrossment

When managing a client's pain during labor, nurses should a. make sure the agents do not prolong labor b. know that all pain relief measures are similar c. support the client's decisions and requests d. not recommend nonpharmacologic methods

c. support the client's decisions and requests

A new parent is alarmed at the shape of the newborn's head. Assessment reveals swelling at the area of the presenting part. The swelling crosses the suture lines. The nurse suspects which condition? molding cephalohematoma closed anterior fontanel (fontanelle) caput succedaneum

caput succedaneum

Patterned breathing techniques used in labor provide which benefits? Select all that apply. spirituality distraction pain relief without special tools conscious relaxation

conscious relaxation distraction pain relief without special tools

When providing prenatal education to a pregnant woman with asthma, which action would be important for the nurse to take? a. explain that the client should avoid steroids during her pregnancy b. demonstrate how to asses her blood glucose levels c. teach correcct administration of subcutaneous bronchodilators d. ensure she seeks treatment for any acute exacerbation

d. ensure she seeks treatment for any acute exacerbation

Which intervention would be least effective in caring for a clinet who is in the active phase of labor? urging the client to focus on one contraction at a time providing one-to-one support encouraging the client to ambulate having the client breathe with contractions

encouraging the client to ambulate

A client who gave birth by cesarean birth 3 days ago is bottle-feeding her neonate. While collecting data the nurse notes that vital signs are stable, the fundus is four fingerbreadths below the umbilicus, lochia are small and red, and the client reports discomfort in her breasts, which are hard and warm to touch. The best nursing intervention based on this data would be: informing the primary care provider that the client is showing early signs of breast infection. having the client stand facing in a warm shower. encouraging the client to wear a supportive bra. using a breast pump to facilitate removal of stagnant breast milk.

encouraging the client to wear a supportive bra.

A nurse is conducting a presentation about prenatal care and preexisting maternal conditions. When discussing the various risks to the mother and infant, the nurse would include information about which condition as the leading cause of intellectual disability in the United States? maternal drug addiction genetic anomalies fetal alcohol spectrum disorder pregnancy category X medications

fetal alcohol spectrum disorder

A pregnant client at 32 weeks' gestation has been admitted to a health care center reporting decreased fetal movement. Which fetal structure should the nurse determine first before auscultating the fetal heart sounds? fetal buttocks fetal back fetal head fetal shoulders

fetal back

A nurse performs an initial assessment of a laboring woman and reports the following findings to the primary care provider: fetal heart rate is 152 bpm, cervix is 100% effaced and 5 cm dilated, membranes are intact, and presenting part is well applied to the cervix and at -1 station. The nurse recognizes that the client is in which stage of labor? first, active third first, latent second

first, active

To give birth to her infant, a woman is asked to push with contractions. Which pushing technique is the most effective and safest? head elevated, grasping knees, breathing out lying on side, arms grasped on abdomen lying supine with legs in lithotomy stirrups squatting while holding her breath

head elevated, grasping knees, breathing out

Thirty minutes after receiving pain medication, a postpartum woman states that she still has severe pain in the perineal region. Upon assessing and palpating the site, what can the nurse expect to find that might be causing this severe pain? infection DVT hematoma nothing—it is normal

hematoma


Ensembles d'études connexes

Chapter 27: The Child with Genitourinary Dysfunction (Peds)

View Set

Retailing & Multichannel Marketing

View Set

(Feist) Chapter 10: Intelligence, Problem Solving, and Creativity

View Set

Burnes - Managing Change Chapter 6 (L4) "MANAGERIAL CHOICE AND CONSTRAINTS"

View Set

Lecture 12. Autonomic Nervous System

View Set

bingham Gerund and participle exam

View Set